SlideShare a Scribd company logo
1 of 29
Download to read offline
1. A 28-year-old, para 2 had a vaginal delivery 20 days ago. She is exclusively breastfeeding
her child. She is sexually active but currently not using a contraceptive method. She is
hoping that lactation amenorrhoea method (LAM) will prevent her getting pregnant. 

Which one of the following will not increase the risk of her getting pregnant?


a. Stopping the night feeds

b. Supplementary feeding

c. Increase in breastfeeding frequency
d. Use of pacifiers

e. Return of menstruation
2. A 28-year-old woman para 4 seeks advice from GP regarding contraception. She had a
vaginal delivery 10 days ago but is happy to use the pills. 

Which of the following statements regarding initiating contraception is correct during the
postpartum period?


a. Progesterone-onlypill(POP)canbesafelystartedpriorto21days 

postpartum.
b. Use of COC between 6 weeks and 6 months is recommended in fully 

breastfeeding women 

c. Non-breastfeeding women can start COC before day 21 postpartum 

d. Postpartum women (breastfeeding and non-breastfeeding) can start the 

POP only after 6 weeks 

e. Breastfeeding women can start COC in the first 6 weeks postpartum as 

there is sufficient evidence to prove the safety of COC use while establishing
breastfeeding
3. Concerning the use of contraception methods in women with cardiac disease, which one of
the following is a correct statement?
a. Prophylactic antibiotics are recommended during insertion or removal of 

intrauterine contraception in women with an increased risk of infective 

endocarditis
b. In women on warfarin therapy the risk of bleeding complication is very 

high during insertion of a progestogen-only implant and therefore its use should be
restricted in these women 

c. The intrauterine device should be fitted in the hospital setting if the risk of
vasovagal reaction is particularly high
d. A causal association has been demonstrated between progestogen-only contraceptive
and venous thromboembolism 

e. The cardiologist should always be involved in deciding whether to use an
intrauterine contraceptive device 

4. A28-year-oldwomanattendsafamilyplanningclinicforcontraceptionadvice. 

She has tested positive for BRCA1 gene. She has blood pressure (BP) of 140/92 at her last
visit with GP. 

The following are true regarding contraceptive use for her except which one?
a. She can be advised that there may be an additional risk of breast cancer with COC
use 

b. She can be advised that there is a reduction in the risk of colorectal cancer with COC
use 

c. In view of her being a BRCA1 gene carrier, her risk of ovarian cancer increases by
30% with COC use 

d. She can be advised that COC use provides a protective effect against endometrial
cancer that continues for 15 years or more after stopping COC 

e. Hypertension may increase the risk of stroke and myocardial infarction (MI) in those
using COC 

5. A 14-year-old girl was having a suction termination of pregnancy (TOP). She did not want
her mother to know. Her mother asks you about why her daughter is in the hospital. 

The options in the girl’s case include the following except for which one? 

a. Disclose to the mother

b. Speak to the patient about disclosure

c. Encourage patient to disclose to mother
d. Advise safe sex practices to patient

e. Advise reliable contraception to patient 

6. One of the following conditions does not fall into UKMEC (UK Medical Eligibility Criteria
for Contraceptive Use) category 1 for using levonorgestrel intrauterine system (LNG-IUS)
a. Infections including past pelvic inflammatory disease (PID) with 

subsequent pregnancy 

b. Schistosomiasis (with fibrosis of the liver) 

c. Non-pelvic tuberculosis or malaria 

d. Infections including past PID without subsequent pregnancy 

e. Superficial venous thrombosis (varicose veins or superficial 

thrombophlebitis)


7. Which one of the following conditions falls into UKMEC category 3 for using a LNG-IUS?

a. Complicated valvular and congenital heart disease

b. Women with known pelvic tuberculosis 

c. Severe dysmenorrhoea
d. Past history of breast cancer with no recurrence in the last 5 years

e. Undiagnosed breast mass or carriers of gene mutations (e.g. BRCA1)
8. A 26-year-old woman para 1 chooses to have a LNG-IUS as a method of contraception. She
has been using other forms of contraception. 

In which of the following scenarios mentioned below will this woman need the use of extra
protection (condoms or abstinence) for 7 days after insertion?


a. If the woman is within <12 weeks since last progestogen-only injection
b. If the woman is within 3 years of insertion of a subdermal implant 

c. If the woman is no later than day 1 of the hormone-free interval for pills or patch 

d. If the woman is within 7 days post-abortion or miscarriage

e. If the woman is partially breastfeeding, amenorrhoeic and less than 

3 months’ postpartum


9. A 20-year-old woman was brought to A&E with a history of sexual assault

4 days ago. The perpetrator had forced her to have vaginal intercourse (used condom) and
also performed digital anal penetration. You are the doctor in A&E who is now collecting
the samples for forensic medical examination. All the following samples are indicated for
forensic medical examination except which one? 

a. Low vaginal swab
b. Vulval swab

c. Peri-anal swab

d. High vaginal swab
e. Anal canal swab 

10. A 15-year-old girl comes to the family planning clinic for contraception. Her last period was
6 weeks ago and a urine pregnancy test shows a positive result. She expresses a strong wish
for termination of pregnancy. 

The following are basic principles for consenting this young person for the procedure except
for which one?
a. A patient can only provide consent if she is deemed to have the capacity to 

consent to treatment. Gillick competence may be applied 

b. A patient having legal capacity may refuse to have treatment for good/bad 

reasons or no reason at all and cannot be compelled to have treatment 

c. Competence/capacity in children below the age of 16 years, for instance, teenage
pregnant women; the degree of capacity required varies with the 

seriousness of the decisions to be taken 

d. That the young person understands the procedure and has the maturity to 

understand what is involved 

e. Consent cannot be revoked by the patient at any time before the treatment 

is given even if she retains the capacity to do so
11. You are the senior registrar on call for gynaecology. A 15-year-old girl attends emergency
department with heavy vaginal bleeding at 16 weeks’ gestation and therefore she is taken to
theatre for removal of productions of conception 

under general anaesthesia. She is haemodynamically unstable and her blood test reveals that
the haemoglobin is 5 gm%. The patient is scared and her parents decline to give consent for
blood transfusion for their daughter despite explaining that she would die. The parents are
Jevohah’s Witness. Her partner is at work.
What should the doctor do in this situation?
a. Call the trust solicitor for advice

b. Call the senior sister in charge to talk to the parents
c. Give blood transfusion to save the girl

d. Listen to the parents

e. Wait for her partner to come
For the following scenarios choose the most appropriate contraceptive method from the list above
A. Copper bearing intrauterine device
B. Levonorgestrelintrauterinesystem
C. Norethisterone5mg
D. Desogestrel 75 mcg
E. IMmedroxyprogesteroneacetate150mg
F. Levonorgestrel 150 mcg and ethinylestradiol 30 mcg
G. Desogestrel 150 mcg and ethinylestradiol 30 mcg
H. Cyproterone acetate 2mg and 35 mcg ethinylestradiol
I. Endometrial ablation
J. Hysterectomy
12) A 43 year old requests contraception. She has 3 children with her ex-husband. She has been in a
new relationship for the past 3 months and has been using condoms. She has no personal or family
history of venous thromboembolism, cancer or heart disease. Her periods are regular occurring
every 28-30 days. She advises you these are very heavy and reports she has tried taking tablets for
these in the past but they didn't help. She has no plans for anymore children. She is a non-smoker.
On examination BP 132/86, HR 84, BMI 30.5 kg/m2
13) The gastroenterology team ask your advice regarding contraception for a 40 year old alcoholic
who has been admitted with decompensated liver cirrhosis. She is having regular unprotected sexual
intercourse. She smokes 20 cigarettes per day. Her BP is 102/60, HR 100 reg, BMI 19.5 kg/m2far).
14) A 39 year old requests contraception. She has been in a new relationship for 3 months. She has
no personal or family history of VTE, cancer or heart disease. Periods are regular every 28-30 days.
She reports these are very heavy, mefanamic acid tablets in the past didn't help. She has no plans for
children in the next few years at least and isn't sure if she wants to have children at all. She is a
non-smoker. Obs: BP 136/73, HR 74, BMI 33.5 kg/m2. Pelvic ultrasound shows 3 small fibroids
measuring 1.0 cm - 1.5 cm and a bicornuate uterus
15) A 22 year-old woman requests contraception. She has tried Microgynon in the past but this
didn't help her acne which bothers her. She has no personal or family history of venous
thromboembolism, cancer or heart disease. Her periods are regular occurring every 28-30 days. She
is a non-smoker. On examination BP 118/68, HR 82, BMI 27.5 kg/m2 and you note moderate facial
acne with some dark hair visible above the upper lip
16) A 24 year old woman requests contraception. She has used condoms in the past but has started a
new relationship and and does not want to use barrier methods. Her cycles are regular every 30 days
and she is a non-smoker. On examination BP 118/68, HR 82 and BMI 27.5 kg/m2 . You note she a
diagnosis of systemic lupus erythematous with the last flare 18 months ago.
17) A 42 year old requests contraception. She has recently started a new relationship and has been
using condoms. She has no personal or family history of venous thromboembolism, cancer or heart
disease. Her periods are regular occurring every 28-30 days. She is a non-smoker. On examination
BP 128/78, HR 76, BMI 30.5 kg/m2
18) A 16 year old patient is seen in family planning clinic as she has been prescribed antibiotics for
a chest infection by her GP. She didn't reveal to her GP that she was taking the combined oral
contraceptive pill as the GP is a family friend and she didn't want the GP to know she was sexually
active. One of her friends has scared her by telling her antibiotics stop contraception from working.
Which of the following antibiotics requires additional contraceptive precautions?
A) Amoxicillin

B) Clarithromycin

C) Doxycycline

D) Ciprofloxacin

E) None of the above
19) A patient wants to use her diaphragm for contraception following delivery of her child. She is
not breastfeeding. When would you advise the patient it is safe to start using this?
A) Can use immediately
B) 3 months post delivery
C) 3 weeks post delivery
D) 4 weeks post delivery
E) 6 weeks post delivery
20) A 23 year old woman asks for your advice regarding contraception. She has tried multiple
combined and progesterone only oral contraceptives but had to discontinue due to mood swings or
heavy bleeding. She doesn't want long acting reversible contraception and doesn't like the feel of
condoms. She is considering the use of a diaphragm with spermicide. What would you advise her
regarding timing of diaphragm removal after sexual intercourse?
A) Remove immediately after intercourse

B) Remove 12 hours after intercourse

C) Remove 6 hours after intercourse

E) Remove 1 hour after intercourse


21) A 22 year old woman asks for your advice regarding contraception. She has tried multiple
combined and progesterone only oral contraceptives but had to discontinue due to mood
swings or heavy bleeding. She doesn't want long acting reversible contraception and doesn't
like the feel of condoms. She is considering the use of a diaphragm with spermicide. What
would you advise her regarding timing of spermicide application prior to sexual intercourse?
A) Spermicide should be reapplied if diaphragm or cap has been in situ for 3 hours or more and sex
is to take place
B) Spermicide should be reapplied if diaphragm or cap has been in situ for 2 hours or more and sex
is to take place
C) Spermicide should be reapplied if diaphragm or cap has been in situ for 1 hour or more and sex
is to take place
D) Spermicide should be reapplied if diaphragm or cap has been in situ for 6 hours or more and sex
is to take place
E) Spermicide should be reapplied if diaphragm or cap has been in situ for 30 minutes or more and
sex is to take place
22) 
A patient wants to restart her combined contraceptive pill following the delivery of her child. She is
not breastfeeding. When would you advise the patient to start taking their contraceptive pill?

A) Start immediately

B) 2 weeks post delivery

C) 3 weeks post delivery
D) 4 weeks post delivery

E) 6 weeks post delivery
23) 
A 24 year old patient is considering starting combined hormonal contraception (CHC). She is
worried about the possibly of thromboembolism having read about some pills being banned in
France. Which of the following has the lowest risk of venous thromboembolism?

A) Ethinylestradiol plus cyproterone acetate

B) Ethinylestradiol plus Norethisterone

C) Ethinylestradiol plus Drospirenone

D) Ethinylestradiol plus Desogestrel

E) Ethinylestradiol plus Gestodene
24) A 22 year old woman asks for your advice regarding contraception. She has tried multiple
combined and progesterone only oral contraceptives but had to discontinue due to mood swings or
heavy bleeding. She doesn't want long acting reversible contraception. She asks about male
condoms. Assuming correct use what would you advise the patient regarding effectiveness of the
male condoms in preventing pregnancy?
A) 88%
B) 92% 
C) 95%

D) 98%

E) 100%
25) Which of the following progestogens is associated with the highest risk of venous
thromboembolism when used with Ethinylestradiol in combine hormonal contraception?

A) Drospirenone

B) Levonorgestrel

C) Norgestimate

D) Etonogestrel

E) Norethisterone
26) 
A patient wants to restart her progesterone only pill following the delivery of her child. She is not
breastfeeding. When would you advise the patient to start taking their contraceptive pill?

A) Advise pill contraindicated

B) Can start immediately

C) 4 weeks post delivery

D) 5 weeks post delivery

E) 6 weeks post delivery
27) 
A 24 year old woman has missed her last 2 combined oral contraceptive pills and had UPSI 24
hours ago. The last pill she took was day 3 of her new pack. What would you advise?
A) Advise her to take both missed pills now then resume normal pill taking.
B) Advise her to take missed pills now, resume normal pill taking with next pill but to run back to
back packs i.e. no pill free break
C) Advise her to take 1 pill now and next pill when due.
D) Advise her to take next pill at normal time and resume pill taking.
E) Emergency contraception
28) 
A patient wants to ask your advice about contraception following delivery of her baby. She is not
breastfeeding and would like the Mirena coil. When would you advise the patient it would be safe
to have this fitted?

A) Anytime

B) 3 weeks after delivery

C) 4 weeks after delivery

D) 6 weeks after delivery

E) 3 months after delivery

29) A 26-year-old woman comes to your clinic requesting emergency contraception. She is not
currently in a stable relationship. She had unprotected intercourse 4 days ago. She is a para 1+2.
Her son is 7 months old and she has had two terminations of pregnancy in the past. She is on day 20
of her cycle and has taken EllaOne® 10 days ago. She has no significant medical or surgical history.
What option of emergency contraception is best suited for her?
a) Copper IUCD insertion and STI risk assessment
b) Intrauterine system insertion
c) Norethisterone tablet
d) Repeat dose of ellaOne®
e) Two tablets of Levonelle®
30) A 15-year-old girl is 10 weeks pregnant and has self-referred requesting termination of
pregnancy.
What is not part of a routine consultation?
a) Check for Gillick’s competency as she is less than 16 years of age
b) Check Rhesus status as she may need anti-D
c) Consider child safeguarding issues by asking about details of partner
d) Explain risks and side effects of procedure
e) Inform parents as would need general anaesthesia for procedure
31) A 35-year-old para3 comes to clinic for contraceptive advice. She was advised to come
off the combined pill as she developed migraines with aura while taking it. She is now
using the diaphragm but is looking for something more reliable.
With regards to duration of action of long-acting reversible contraception, which one of
these statements is not correct?
a) Depo-Provera® injection is effective for 13 weeks
b) Mirena coil is effective for 5 years
c) Multiload copper coil is effective for 5 years
d) Nexplanon® is effective for 5 years
e) T-Safe® Cu380A is effective for 10 years
32) A 27-year-old nulliparous woman comes to the outpatient clinic requesting Depo-
Provera® injection. She wants to know about the non-contraceptive benefits and risks with
injectable progesterones.
Which of the following is incorrect?
a) Anaemic women may benefit from using injectable progestogens
b) Depot medroxy progesterone acetate is associated with increase in bodyweight
c) Injectable contraceptives have been associated with an increased risk of developing
venous thromboembolism in women with a previous history of venous thromboembolism
after immobilisation
d) It can reduce pain due to endometriosis
e) Women with sickle cell disease may notice a decrease in pain during sickle cell crisis
33) You are going to prescribe combined transdermal patches for a 15 year old girl with
cerebral palsy as she is unable to cope with her periods. For the combined transdermal
patches to achieve the same effect as continuous oral contraceptive pill, for how long the
patches should be applied serially for?
a) 3 weeks followed by 7-day withdrawal period
b) 4 weeks followed by 7-day withdrawal period
c) 5 weeks followed by 7-day withdrawal period
d) 7 weeks followed by 7-day withdrawal period
e) 9 weeks followed by 7-day withdrawal period
34) A 30-year-old woman had a copper intrauterine device inserted immediately after she
had a normal vaginal delivery. She presents 3 weeks later because she cannot feel the
thread. What would be the most appropriate step to take in her management?
a) Perform a pregnancy test and then arrange an ultrasound scan
b) Perform a pelvic examination first, then a pregnancy test, and then an ultrasound scan if
the test is negative
c) Perform an ultrasound scan and if the device is not seen, request a urine pregnancy test
d) Perform an ultrasound scan and if the device is not located, perform a serum hCG
assay
e) Perform a pelvic examination and if the thread cannot be seen, perform an ultrasound
scan and if negative, perform a pregnancy test
35) In which group of women with a heart disease undergoing an IUD insertion does the
FSRH recommend the administration of prophylactic antibiotics?
a) Those with a history of rheumatic fever
b) Those with a prosthetic valve
c) Those with congenital heart disease
d) Those with hypertrophic cardiomyopathy
e) Those with septal defects
36) A 30-year-old attends for contraceptive advice. She suffers from pulmonary
hypertension and does not wants to get pregnant. What WHOMEC (UKMEC) category will
you place this patient in with respect to offering her contraception?
a) UKMEC category I
b) UKMEC category II
c) UKMEC category III
d) UKMEC category IV
e) UKMEC category V
37) Which progestogen in combined hormonal contraception is associated with the highest
risk of venous thromboembolism?
a) Gestodene
b) Levonorgestrel
c) Medroxyprogesterone
d) Norethisterone
e) Norgestimate
38) A 45-year old is on the progestogen-only pill containing desogestrel. What would be
considered a missed pill in this woman?
a) Delay in taking the pill by more than 3 hours
b) Delay in taking the pill by more than 6 hours
c) Delay in taking the pill by more than 12 hours
d) Delay in taking the pill by more than 8 hours
e) Delay in taking the pill by more than 24 hours
39) When counselling a man regarding vasectomy, which of the following is it important to
stress
a) Contraception should be used for a further 3 months with a negative semen analysis at
the end of that time
b) Reversal is readily available as an NHS procedure
c) The procedure is irreversible
d) There is a small increased risk of testicular cancer following the procedure
e) Vasectomy has a higher failure rate than female sterilization
40) Which of the following statements is the most appropriate concerning long-acting
reversible contraception and intrauterine devices (IUDs)
a) Gynefix is currently the only frameless IUD licensed in the UK.
b) IUDs available in the UK are licensed for a variable time period from 5 to 8 years.
c) The most effective IUDs contain at least 280 mm2 of copper
d) The risk of ectopic pregnancy increases in women using IUDs
e) Unbanded devices are the most effective IUD presently available
41) Concerning LARC injectables and implants, below are true EXCEPT:
a) The major mechanism of action of depomedroxyprogesterone acetate (DMPA) is to
thicken cervical mucus.
b) Progestogen implants are associated with irregular vaginal bleeding.
c) The failure rate of progestogen implants is higher in women with a BMI 40.
d) DMPA does adversely affect bone density in long-term use and, therefore, it is not
recommended as a first-line contraceptive in young women.
e) Approximately 50% of women in the UK discontinue DMPA within 1 year
42) Concerning progestogen-only pills (POPs), which of the following is true?
a) Additional contraceptive precautions should be taken during antibiotic treatment
b) In women undergoing treatment for breast cancer, POPs increase the rate of disease
recurrence
c) They are associated with an increased risk of thromboembolic disease
d) They are contraindicated in women with insulin-dependent diabetes
e) They should be discontinued 4 weeks prior to major surgery
43) When counselling a woman for postcoital contraception (PCC), what should you
advise
a) Intrauterine PCC can be inserted up to Day 19 of a 28-day cycle.
b) Mifepristone can be used for PCC
c) STI screening results should be available prior to fitting intrauterine PCC
d) The intrauterine system is licensed for PCC
e) Ulipristal acetate is as effective as an IUD
44) An 18-year-old p1+0 (1-year-old child, normal vaginal delivery) attends her local sexual
reproductive health clinic requesting contraception. Her first pregnancy was unplanned
and she wants a reliable method of contraception, at the same time planning for a further
pregnancy in 12–18 months.
She is in a steady relationship and has been with the same partner for 2 years. Her
periods are irregular, heavy and painful, and have got worse over recent months, with a
cycle of 5–7/21–38 days. She has also noted increased facial hair, requiring electrolysis.
She has previously been diagnosed with PCOS. She has no relevant family history. On
examination she looks well:
• height: 153 cm
• weight: 94 kg
• BP: 120/75 mmHg.
A: Condoms
B: Levonorgestrel IUS (Mirena®)
C: Depo medroxyprogesterone acetate (Depo-Provera®)
D: Noriday® progestogen-only pill
E: Dianette® combined oral contraceptive pill
Choose the method of contraception that is the most suitable for her
45) Miss Y is a 22-year-old nulliparous young woman who presents at her local sexual
reproductive health clinic having had unprotected sexual intercourse on day 16 of a regular
28-day cycle. She is in a new relationship, having had intercourse last night with her 27-
year-old boyfriend who she met last week at a party.
She has not had any other episodes of unprotected intercourse this cycle. She suffers
from epilepsy and takes carbamazepine 400 mg twice daily. She is otherwise well with no
relevant family history. She looks well and her measurements are as follows:
• height: 159 cm
• weight: 58 kg
• BP: 115/65 mmHg.
A: Insert a copper IUD as postcoital contraception and continue to use the IUD as her
preferred method of contraception
B: Levonelle 1.5 mg immediately as postcoital contraception, repeated after 12 hours.
Condoms and commence Depo-Provera® to be administered every 12 weeks at the onset
of her next period
C: Levonelle 1.5 mg immediately as postcoital contraception, repeated after 12 hours.
Condoms and commence Cerazette® progestogen-only pill at the onset of her next period
D: Levonelle 1.5 mg immediately as postcoital contraception. Condoms and commence
Depo-Provera® to be administered every 10 weeks at the onset of her next period
E: Levonelle 1.5 mg immediately as postcoital contraception. Condoms and commence
Microgynon 30 combined oral contraceptive at the onset of her next period
Choose the method of contraception that is the most suitable for her
46) Mrs P, a 32-year-old para 4+0, has just delivered a baby boy (SVD) following an
unplanned pregnancy, having recently separated from her husband and entered into a new
short-term relationship. She is a week postpartum and requesting a reliable form of
contraception.
She has a family history of VTE in a first-degree relative and is known to carry a protein S
deficiency but has never had a VTE episode herself. She is bottle feeding, her puerperium
has been uncomplicated and she is keen to start contraception as soon as possible.
• Height: 160 cm
• Weight: 65 kg
• BP: 135/80.
A: Advise regarding STI and condom use, follow-up appointment to discuss male or female
sterilisation
B: Immediate insertion of copper IUD (T-Safe® Cu380A), advise regarding STI and
condom use
C: Commence progestogen-only pill (Cerazette®) immediately, advise regarding STI and
condom use
D: Commence combined oral contraceptive (Microgynon® 30) 4 weeks postpartum, advise
regarding STIs and condom use
E: Advise regarding STI and condom use, and injection of Depo-Provera® after 6 weeks
Choose the method of contraception that is the most suitable for her
47) A 35-year-old para 1+1 has recently undergone surgical management of miscarriage
following a proven molar pregnancy. The hCG levels are being monitored and are still
significantly elevated. She is continuing to experience irregular vaginal bleeding following
the evacuation of the uterus.
On examination:
• height: 156 cm
• weight: 58 kg
• blood pressure: 110/65 mmHg
She is in a long-term relationship and is requesting contraception.
A: Combined hormonal pill containing 30 microgram ethinylestradiol and a first-generation
progestogen
B: Levonorgestrel IUS (Mirena®)
C: Depo-medroxyprogesterone acetate (Depo-Provera®)
D: The desogestrel progestogen-only pill
E: Subdermal implant
Which method of contraception is contraindicated?
48) A 26-year-old woman comes to your clinic requesting emergency contraception. She is
not currently in a stable relationship. She had unprotected intercourse 4 days ago. She is a
para 1+2. Her son is 7 months old and she has had two terminations of pregnancyin the
past. She is on day 20 of her cycle and has taken EllaOne® 10 days ago. She has no
significant medical or surgical history.
What option of emergency contraception is best suited for her?
a) Copper IUCD insertion and STI risk assessment
b) Intrauterine system insertion
c) Norethisterone tablet
d) Repeat dose of ellaOne®
e) Two tablets of Levonelle®
49) A 35-year-old para3 comes to clinic for contraceptive advice. She was advised to come
off the combined pill as she developed migraines with aura while taking it. She is now
using the diaphragm but is looking for something more reliable.
With regards to duration of action of long-acting reversible contraception, which one of
these statements is not correct?
a) Depo-Provera® injection is effective for 13 weeks
b) Mirena coil is effective for 5 years
c) Multiload copper coil is effective for 5 years
d) Nexplanon® is effective for 4 years
e) T-Safe® Cu380A is effective for 10 years
.
50) A 26-year-old para 1 comes to your clinic requesting contraception. She mentions that
her maternal aunt had deep vein thrombosis (DVT) and is on warfarin. She is worried that
if she takes hormonal contraception she will also develop DVT.
Which of the following advice regardling DVT is incorrect.
a) Combined oral contraceptives containing a third generation progestogen is associated
with an increased risk of venous thromboemolism (VTE) when compared with those
containing second generation progestogens
b) The risk of VTE increases with the number of years of usage of a combined hormonal
pill
c) There is little or no data to suggest that there is an increased risk of VTE with oral or
injectable progesterone only contraceptives methods
d) Thrombophillia screen is not recommended routinely before use of combined hormonal
contraceptive
e) Observational studies have reported that the transdermal patch has a somewhat
similar level of VTE risk to combined oral contraceptives containing second-generation
progestogens
51) A couple have come to your clinic requesting sterilisation. After going through the risks
and benefits of the procedures they opt for vasectomy.
Which of the following should not be taken into consideration during vasectomy?
a) 12-week postvasectomy is considered an optimal timing for scheduling first
postvasectomy semen analysis
b) Clips are not recommended for occlusion of the vas deferens as it is associated with a
higher failure rate
c) If motile sperms are found in a fresh semen sample 3 months postprocedure, the
vasectomy is be considered a failure
d) Routine histological examination of the vas deferens is no longer routinely
recommended
e) Warming local anaesthetic to body temperature before infiltration reduces pain due to
injection
52) You are a ST5 and are currently in a minimal access surgery placement. Your
consultant is training you in performing hysteroscopic sterilisation on a 38-year-old woman.
She has had three previous normal vaginal deliveries and a history of Chlamydia
trachomatis infection at age 19; otherwise no other history of note. At the end of the
procedure you are asked to arrange a follow-up ultrasound scan in 3 months' time in order
to confirm occlusion of the fallopian tubes.
Where in the fallopian tubes would the micro-inserts be placed?
a) Ampullar
b) Infundibulum
c) Intraluminal
d) Intramyometrial
e) Isthmus
53) Which of the following is correct concerning Nonoxynol-9
a) Regular use decreases risk of sexually transmitted HPV that can cause cervical cancer
b) Regular use protects against sexually transmitted infections such as gonorrhoea and
chlamydia
c) Regular use reduces the risk of HIV transmission
d) Condoms lubricated with Nonoxynol - have a longer shelf life than those without
e) Is a type of organic surfactant and an active constituent of spermicide
54) A 47 year old woman seeks advice about continuing the combined oral contraceptive
pill ( COCP ) . She is normotensive and a non smoker with a BMI of 25. She has no other
medical history and no significant family history. She is concerned that the COCP may give
her additional health risks. Which of the following malignancies would you advice she may
have a small additional risk of developing due to taking the COCP
a) breast cancer
b) colorectal cancer
c) endometrial cancer
d) lung cancer
e) ovarian cancer
55) A 26 year old woman presents to the emergency gynaecology clinic requesting
emergency contraception ( EC) . She had unprotected sex 6 days ago. She is not currently
using any contraception, having not had a partner for a year. She has a regular 28 day
menstrual cycle, which can be heavy. The first day her last period was 15 days ago.What
emergency contraception option, if any, would you advise?
a) A copper bearing intrauterine device
b) A MIRENA coil
c) It is too late for EC
d) Levonelle
e) Ulipristal acetate
56) A 25 year old woman with a bicornuate uterus attends the emergency gynaecology unit
requesting EC. She has been on holiday and forgot to take her contraceptive pill for 3 days
in the first week of the calendar pack and had unprotected sexual intercourse ( UPSI ) four
days ago. She is in good health. Which of the following is the recommended EC?
a) Copper IUCD
b) MIRENA IUS
c) Levonorgestrel
d) Mifepristone
e) Ulipristal acetate
57) What is the risk of VTE in women using combined hormonal contraception?
a) 1 per 10,000
b) 2-4 per 10,000
c) 5-12 per 10,000
d) 15 - 20 per 10,000
e) 25 - 30 per 10,000
58) When is the risk of VTE highest after starting combined hormonal contraception?
a) In the first one month
b) In the first 4 month
c) In the first year
d) In the first 3 years
e) Remains the same while used
59) A 34 year old woman has an elective caesarean section because she is HIV positive
She is taking anti-retroviral drugs and wishes to use depo-medroxyprogesterone acetate
for contraception. Depo - medroxyprogesterone acetate can be used without restriction
( UKMEC 1) in women taking which anti- retroviral drugs?
a) Indanivir
b) Nevirapine
c) Lamivudine
d) Zidovudine + Efavirenz
e) All the drugs listed
60) A 34 year old woman attends the emergency gynaecology clinic requesting emergency
contraception. her LMP was 2 weeks ago and she has a regular 28 day cycle. She had
unprotected intercourse 24 H earlier. She suffers from epilepsy and is treated with
phenytoin. Which one is the most appropriate option?
a) Levonorgestrel 1500mcg x 2 doses 12 hours apart
b) Levonorgestrel 750mcg x 2 doses 12 hours apart
c) Ethinyl-oetradiol 500mcg
d) Levonorgestrel 1500mcg followed by 750mcg after 12 hours
e) Ethinyl-oestradiol 100mcg + levonorgestrel 750mcg
61) A 23 years old nulliparous woman attends AE. She gives a history of UPSI the
previous night. Her periods are regular 3-4/30. She had sexual intercourse with a stranger.
Her last menstrual period was 2 weeks ago. What is the most appropriate step?
a) Levonorgestrel 2 tablets stat and UPT in 2 weeks time if there has been no period
b) Reassurance since UPT is negative and to cont male condom
c) Levonorgestrel 2 tablets stat, UPT in 2 weeks time if there has been no period and
offer STI screening
d) Copper IUCD stat and UPT in 2 weeks time if there has been no period
e) Levonorgestrel - releasing intrauterine system as benefit for long term contraception
62) A 17 year old girls requests EC. She gives a history of unprotected sexual intercourse
40 hours ago. She is unsure of when her LMP was as her period are irregular. Her regular
contraceptive method is the male condom. What is the next management ?
a) Sustain further history about the partner ( age, relationship, promiscuity)
b) Offer screening for STD
c) Assess Gillick/Fraser competency
d) Azithromycin stat dose
e) Pregnancy test now and may need to be repeated later
63) A 25 year old woman requests emergency contraception following UPSI on one
occasion 4 days ago. She is currently taking a proton pump inhibitor for gastro-
oesophageal reflux. What EC would you recommend?
a) Copper IUCD
b) Levonorgestrel
c) MIRENA
d) Mifepristone
e) Ulipristal acetate
64) The United Kingdom Medical Eligibility Criteria ( UKMEC) offers guidance to clinicians
when considering the use of contraceptives. There are 44 categories, with category 1
being generally suitable and Category 4 completely contraindicated. when considering the
combined oral contraceptive pill, which of the following factors would be considered as a
Category 4( completely contraindication )?
a) Age 30 and smoking ( more than 35/day)
b) Family history of breast cancer with a known BRCA gene mutation
c) History of DVT
d) Hypertension 140/90
e) Three weeks postnatal and breastfeeding
65) A 34 year old woman with BMI of 34 presents for contraceptive advice. She has had
treatment for DVT. Concerning hormonal contraception and thromboembolic disease,
which of the following is true?
a) For women who are post part and not breast feeding , combined hormonal
contraception can be started immediately
b) for women with current VTE on anticoagulants, the POPs is contraindicated
c) Hormonal contraception can be started immediately after a 1st and 2nd trimester TOP
d) POPs, implants and MIRENA have an increased risk of VTE
e) Women with factor V leiden have up to a five-fold increase in the risk of VTE with
COCP use
66) A 24 year old woman attends your clinic for EC after 2 episodes of UPSI over the last
24 hours. She is currently on treatment for tuberculosis. Which is the most appropriate
contraceptive to prescribe her?
a) Copper IUD
b) Implanon
c) Levonorgestrel
d) MIRENA
e) Ulipristal acetate
67) A 33 yo woman consults you for contraception following her second delivery. She does
not want oral medication and asks about the combined vaginal ring. Which of the following
is correct?
a) it contains ethynylestradiol and levonorgestrel
b) it is changed every 6 months
c) it is uneffected by enzyme inducing drugs
d) It should be kept refrigerated before use
e) The risk of VTE is reduced when compared with COC
68) A 26 year old woman with known case of heavy menstrual bleeding would like to try on
MIRENA coil . She also keen for LARC. What is common risk associated with the MIRENA
coil ?
a) Expulsion, occuring in 1% of cases
b) Expulsion , occuring in 5% of cases
c) Failed contraception, occurring in 1% of cases
d) Perforation, occuring in 1% of cases
e) Perforation, occuring in 5% of cases.
69) You review a 17 - year old woman who requests emergency contraception after four
episodes of unprotected sexual intercourse over the last five days. She is asthmatic, with
an admission to ITU last year. Which method of contraception is the most appropriate to
prescribe?
a) Copper intrauterine device
b) Levonorgestrel pill
c) Mirena coil
d) Progesterone - only pill
e) Ulipristal acetate
70) A 32-year-old woman comes to see you following her second pregnancy, which
resulted following contraceptive failure. All the following contraceptive methods primarily
work by inhibiting ovulation, except
a) Combined hormonal contraceptive pill (CHC)
b) Combined vaginal ring ( CVR)
c) Depot medroxyprogesterone acetate ( DMPA)
d) Levonorgestrel-containing intrauterine system ( IUS)
e) Subdermal contraceptive implant
71) You discuss ulipristal acetate ( UPA) - containing emergency contraception for a 24-
year-old woman requesting emergency contraception. Which of the following is correct?
a) A double dose should be given if the woman is using liver enzyme-inducing medication
b) It can be administered beyond 120 hours
c) It is considered first-line for women taking antacids or H2 receptor antagonists
d) It should only be used once in a menstrual cycle
e) The primary mechanism of action is prevention of implantation
72) A 40 year- old on the COCP wishes to discuss the benefits of continuing with the pill
with regards to ovarian cancer protection. What advice will you offer her?
a) COCP reduce the risk of ovarian cancer by approximately 25%
b) Low dose COCP (  35ug ethinyl estradiol) do not confer adequate protection
compared to high dose COCP
c) The decrease risk for ovarian cancer from COCP use is independent of use after the
first 12 months
d) The protective effect of COCP on ovarian cancer risk ceases 5-10 years after stopping
the pills
e) Women who have BRCA 1 and BRCA 2 mutations should not go on the COCP
73) A 33 year- old healthy lady , para 2 missed to take 3 pills of her COC ( pills day 5 - day
7). She had a sexual intercourse 48 hours ago.
a) Emergency contraception
b) Take missed pills as soon as possible, continue remaining pills and emergency
contraception
c) Take her 2 missed pills as soon as possible, continue remaining pills and emergency
contraception
d) Take recent missed pills as soon as possible, continue remaining pills, emergency
contraception and additional contraceptive for 7 days
e) Take recent missed pills as soon as possible, continue remaining pills, no need
emergency contraception but needs additional contraceptive for 7 days
74) 25 year-old healthy lady, Para 1 missed to take 4 pills of her COCP on day 18 - day 21.
She presented to you after she had sexual intercourse 48 hours ago.
a) Take recent missed pills as soon as possible, continue remaining pills, emergency
contraception and additional contaceptive for 7 days
b) Omit pill free interval and start new pack the next day
c) No additional contraception needed
d) Emergency contraception and additional contraceptive for 7 days
e) UPT if no menses in next cycle
75) A 40 years old woman requested for contraception. She is keen for CHC.
Disadvantages of CHC explained to her. Which of the following is not true?
a. Increase risk of breast cancer
b. Increased risk of colorectal carcinoma
c. Reduced risk of benign breast disease
d. Increased risk of cervical cancer
e. Increased risk of ischeamic stroke
EMQ
The following clinical scenarios describe situations in which the use of one or more
contraceptive methods is not advisable. For each scenario, decide which contraceptive
method or methods should be avoided from the list of available options; assume that there
are no other factors that influence contraceptive suitability. Each option may be used once,
more than once, or not at all
1. A 30- year old nulliparous woman with systemic lupus erythematous ( SLE ) with
positive or unknown antiphospholipid antibodies
2. A 26-year-old woman with chronic viral hepatitis with normal liver function.
3. A 27-year-old nulliparous woman with a history of an ectopic pregnancy managed by
salpingectomy.
A All hormonal contraceptive methods, excluding the levonorgestrel-containing intrauterine
system ( IUS)
B All hormonal contraceptive methods, including the levonorgestrel-containing intrauterine
system ( IUS)
C Combined hormonal contraception ( CHC) , combined vaginal ring ( CVR) , combined
transdermal patch ( CTP)
D Combined hormonal contraception ( CHC) and progestogen-only injectables ( depo-provera)
E Copper-containing intrauterine contraceptive device ( IUD )
F Copper-containing intrauterine contraceptive device (IUD) and progestogen-only pill ( POP)
G Levonorgestrel-containing intrauterine system ( IUS)
H Levonorgestrel-containing intrauterine system ( IUS) and copper-containing intrauterine device
( IUD )
I Progestogen-only implants
J Progestogen-only injectables ( depo-provera)
K Progestogen-only pill ( POP)
L Progestogen-only pill ( POP) and progestogen-only implants
M All of the above
N None of the above
Match the most appropriate management to each scenario:
4. A 43-year-old woman started a combined oral contraceptive pill three months ago. She
has started having irregular bleeding. She is known to have PCOS ( polycystic ovarian
syndrome). She is a non-smoker and is up to date with her smears.
5. A 24- year-old woman presents with irregular bleeding six months after starting the
combined oral contraceptive pill. She has no medical problems. She has recently started a
new relationship. She is up to date with her smears.
6. A 50-year-old woman has used the levonorgestrel IUS ( intrauterine system) for heavy
menstrual bleeding for the past five years and is amenorrhoiec. She now wonders whether
she has gone through the menopause, and therefore can have the coil removed.
7. A 52-year-old woman had a copper coil fitted at the age of 43. Her last menstrual period
was 13 months ago. She wishes to know whether she has gone through the menopause
and therefore can have the coil removed.
A Leave in for further 12 months and then remove or check two FSH levels, six weeks apart, and
if both over 30 IU/L, remove
B Take a full sexual history and commence a 30 mug or 35 mug combined oral contraceptive pill.
If not resolving, consider endometrial biopsy
C Take a full sexual history and investigate for sexually transmitted infections if appropriate
D Take a full sexual history and cervical smear
E Stop the contraceptive and advise trying a long-acting reversible contraceptive such as the
implant
F Take a full sexual history, do a pelvic examination and cervical smear. If normal, consider adding
mefenamic acid alongside
G Remove the device now
H Leave in for a further 24 months and then remove or check two FSH levels six weeks apart and
if both over 30 IU/L , remove after 12 months
A Natural family planning
B Male condoms
C Female condoms
D Diaphragm
E Combined oral contraceptive pill
F Evra
G Nuva Ring
Listed are recognised methods of contraception. From this list, select the single
contraception method that is clinically most appropriate given the clinical scenario
described.
10. A 30-year-old woman who continuously forgets to take contraception. She wishes long
intervals between needing to renew her contraception. She is nulliparous in a stable
relationship. She is not keen for anything to be inserted into her uterus.
11. A 40-year-old smoker with a BMI of 40 and has 3 previous caesarean sections. She
feels she has completed her family and wants permanent contraception.
12. An 18-year-old woman with painful periods which are irregular. She is keen on
contraception, preferably something that will regulate her menses. She is nulliparous. She
is not keen to take pills and is scared of needles. She does not want any invasive form of
contraception.
13. A 48-year-old woman with irregular cycle heavy menstrual bleeding. She is fed up with
using POP for contraception. She is in a stable relationship and her family is complete.
She is considering HRT when menopause occurs.
14. A 24-year-old with strong family history of ovarian cancer. Has had one child and is
planning to have bilateral oopherectoy following birth of second child. She seeks effective
contraception that would be beneficial to her long term health.
15. A 25-year-old Para 1 , known case of Rheumatoid arthritis in remission, presented to
your clinic to discuss reliable long acting reversible contraception.
H progestogen only pill
I MIRENA
J Copper IUD
K Depot medroxyprogesterone acetate
L Implanon
M Hysteroscopic sterilisation
N Laparoscopic sterilisation
A Natural family planning
Match the most appropriate advice to each scenario.
16. A 28-year-old woman misses two consecutive 30mcg combined pills in the second
week of taking her packet ( days 9 and 10) . She has been taking all her pills on days 1-7.
She had sexual intercourse with her boyfriend on day 8 .
17. A 30-year- old Para 3, missed to take 1 pill of her COC on pills day 6. She presented to
you after she had sexual intercourse 48 hours ago.
A Take both missed pills, take the remaining pills as usual, condoms should be used for the next
seven days or sexual intercourse avoided in case further pills are missed. No indication for
emergency contraception
B Take the most recent missed pill, take the remaining pills as usual, condoms should be used for
the next seven days or sexual intercourse avoided in case further pills are missed, No
indication for emergency contraception
C Take both missed pills, take the remaining pills as usual , condoms should be used for the next
seven days or sexual intercourse avoided in case further pills are missed. Emergency
contraception should also be advised.
D Take the most recent missed pill, take the remaining pills as usual, condoms should be used for
the next seven days or sexual intercourse avoided in case further pills are missed, Emergency
contraception should also be advised
E Omit pill free interval and start new pack the next day
F Administer the next injection, offer emergency contraception, additional contraception or
abstinence should be advised for the next seven days and a pregnancy test should be carried
out in 21 days
G The next injection should not be administered, emergency contraception should be offered and
a pregnancy test should be carried out in 21 days
H Administer the next injection, emergency contraception not required, additional contraception or
abstinence for the next seven days
I Prescribe additional contraception and start new ring immediately without a ring free interval
J Start new ring immediately without a ring free interval
K Remove ring and wait for withdrawal bleed, if no menses for UPT testing
L Apply new patch now and delay change by 48 hours
M Apply new patch now but keep the same date of change
N Apply new patch and use barrier contraception for 7 days
O Take missed pills as soon as possible, take next pill at usual time, additional contraceptive for 2
days, offer emergency contraception
P Reassure
18. A 36-year-old woman phones for advice because she has missed her contraceptive
pill. Her LMP was 12 days ago and she has regular 28 day cycle. She is using a standard
progestogen-only pill and has missed her one pill by 8 hours. She had unprotected
intercourse 6 hours earlier.
19. A 36-year-old woman phones for advice because she has missed her contraceptive
pill. Her LMP was 12 days ago and she has a regular 28 day cycle. She is using Cerazette
and has missed her one pill by 8 hours. She had unprotected intercourse 6 hours earlier.
20. A 32-year -old woman had her last DMPA ( depo-provera) injection 14 weeks and two
days ago. She had sexual intercourse with her partner two days ago.
21. A 34-year-old Para 3 used combined vaginal ring as her contraception of choice, she
already wearing the ring for 32 days and planned to remove it. Best advice is;
22. A 36-year-old woman on day 8 of her regular menstrual cycle has not changed the
patch for 48 hours.
23. A 25-year-old healthy woman day 30 post partum currently started to have her menses
back and she had take COCP at day 4 of her menses.
24. A 30-year-old lady now day 40 post partum and has her period of amenorrhoea since
delivery took estradiol valerate as her contraceptive method at day 40 post partum.
A Start at Day 1 menses
B Start at Day 5 menses
C No additional contraceptive protection required
D Require additional contraceptive protection and avoid sex for 7 days
E Require additional contraceptive protection and avoid sex for 9 days
F Discard the pills and restart back from day 1 menses
G Discard the pills, induce bled with progesterone and take the pill at day 1 menses
H Avoid sex for 7 days and condom
I Continue pill and contraception for 7 days ( barrier method )
J Offer levonorgestrel 1.5mg stat
25. A 27-year-old healthy lady had first trimester abortion and she took the COCP at day 5
post partum abortion.
26. A 32-year-old healthy lady had recently second trimester miscarriage and she took
COCP at day 20 post abortion. Her UPT test done negative.
27. A 30-year-old healthy lady had her menses on day 40 post partum. She took estradiol
validate as her choice of contraception on the day itself. She presented to you for advice
regarding contraception covers as she is now worried after an SI last night.

More Related Content

Similar to contraception qs1pdf.pdf

Medical Management of Ectopic Pregnancy
Medical Management of Ectopic PregnancyMedical Management of Ectopic Pregnancy
Medical Management of Ectopic PregnancyApollo Hospitals
 
Abortion 2.pptx
Abortion 2.pptxAbortion 2.pptx
Abortion 2.pptxmekdi3
 
Management of IVF Pregnancy
Management of IVF PregnancyManagement of IVF Pregnancy
Management of IVF PregnancyKaberi Banerjee
 
Elective Abortion HE-230-OL
Elective Abortion HE-230-OL Elective Abortion HE-230-OL
Elective Abortion HE-230-OL Meghan George
 
Contraception and its ethical considerations
Contraception and its ethical considerationsContraception and its ethical considerations
Contraception and its ethical considerationsDr Tarique Ahmed Maka
 
2_2019_02_23!1ehefguuthutyj0_34_21_PM.pptx
2_2019_02_23!1ehefguuthutyj0_34_21_PM.pptx2_2019_02_23!1ehefguuthutyj0_34_21_PM.pptx
2_2019_02_23!1ehefguuthutyj0_34_21_PM.pptxJoebest8
 
Current Point of View in Preterm Labor Management in Albania
Current Point of View in Preterm Labor Management in AlbaniaCurrent Point of View in Preterm Labor Management in Albania
Current Point of View in Preterm Labor Management in AlbaniaRustem Celami
 
Pp module content outline
Pp module content outlinePp module content outline
Pp module content outlinenpenwill
 
Update on contraception
Update on contraceptionUpdate on contraception
Update on contraceptiondrajaysd
 
cervical cancer presentation
cervical cancer presentation cervical cancer presentation
cervical cancer presentation alaaag
 
2023 recommendations for Adolescent PCOS.pptx
2023 recommendations for Adolescent PCOS.pptx2023 recommendations for Adolescent PCOS.pptx
2023 recommendations for Adolescent PCOS.pptxDrPushpaSankhwar
 
Adolescent PCOS 2023.pptx
 Adolescent PCOS 2023.pptx Adolescent PCOS 2023.pptx
Adolescent PCOS 2023.pptxDrPushpaSankhwar
 
Current point of view in preterm labor management in albania (2)
Current point of view in preterm labor management in albania (2)Current point of view in preterm labor management in albania (2)
Current point of view in preterm labor management in albania (2)Alexander Decker
 
PELVIC INFLAMMATORY DISEASE (PID)
PELVIC INFLAMMATORY DISEASE (PID)PELVIC INFLAMMATORY DISEASE (PID)
PELVIC INFLAMMATORY DISEASE (PID)Dawood Al nasser
 

Similar to contraception qs1pdf.pdf (20)

Prolonged pregnancy
Prolonged pregnancyProlonged pregnancy
Prolonged pregnancy
 
Hormonal FPM.pptx
Hormonal FPM.pptxHormonal FPM.pptx
Hormonal FPM.pptx
 
Medical Management of Ectopic Pregnancy
Medical Management of Ectopic PregnancyMedical Management of Ectopic Pregnancy
Medical Management of Ectopic Pregnancy
 
Abortion 2.pptx
Abortion 2.pptxAbortion 2.pptx
Abortion 2.pptx
 
Management of IVF Pregnancy
Management of IVF PregnancyManagement of IVF Pregnancy
Management of IVF Pregnancy
 
Combined oral contraceptive pills
Combined oral contraceptive pillsCombined oral contraceptive pills
Combined oral contraceptive pills
 
Contraception
ContraceptionContraception
Contraception
 
Elective Abortion HE-230-OL
Elective Abortion HE-230-OL Elective Abortion HE-230-OL
Elective Abortion HE-230-OL
 
Contraception and its ethical considerations
Contraception and its ethical considerationsContraception and its ethical considerations
Contraception and its ethical considerations
 
2_2019_02_23!1ehefguuthutyj0_34_21_PM.pptx
2_2019_02_23!1ehefguuthutyj0_34_21_PM.pptx2_2019_02_23!1ehefguuthutyj0_34_21_PM.pptx
2_2019_02_23!1ehefguuthutyj0_34_21_PM.pptx
 
Current Point of View in Preterm Labor Management in Albania
Current Point of View in Preterm Labor Management in AlbaniaCurrent Point of View in Preterm Labor Management in Albania
Current Point of View in Preterm Labor Management in Albania
 
Pp module content outline
Pp module content outlinePp module content outline
Pp module content outline
 
Update on contraception
Update on contraceptionUpdate on contraception
Update on contraception
 
Contraceptive.pptx
Contraceptive.pptxContraceptive.pptx
Contraceptive.pptx
 
cervical cancer presentation
cervical cancer presentation cervical cancer presentation
cervical cancer presentation
 
2023 recommendations for Adolescent PCOS.pptx
2023 recommendations for Adolescent PCOS.pptx2023 recommendations for Adolescent PCOS.pptx
2023 recommendations for Adolescent PCOS.pptx
 
Adolescent PCOS 2023.pptx
 Adolescent PCOS 2023.pptx Adolescent PCOS 2023.pptx
Adolescent PCOS 2023.pptx
 
Current point of view in preterm labor management in albania (2)
Current point of view in preterm labor management in albania (2)Current point of view in preterm labor management in albania (2)
Current point of view in preterm labor management in albania (2)
 
PELVIC INFLAMMATORY DISEASE (PID)
PELVIC INFLAMMATORY DISEASE (PID)PELVIC INFLAMMATORY DISEASE (PID)
PELVIC INFLAMMATORY DISEASE (PID)
 
Family planning
Family planningFamily planning
Family planning
 

Recently uploaded

Painted Grey Ware.pptx, PGW Culture of India
Painted Grey Ware.pptx, PGW Culture of IndiaPainted Grey Ware.pptx, PGW Culture of India
Painted Grey Ware.pptx, PGW Culture of IndiaVirag Sontakke
 
Roles & Responsibilities in Pharmacovigilance
Roles & Responsibilities in PharmacovigilanceRoles & Responsibilities in Pharmacovigilance
Roles & Responsibilities in PharmacovigilanceSamikshaHamane
 
Introduction to AI in Higher Education_draft.pptx
Introduction to AI in Higher Education_draft.pptxIntroduction to AI in Higher Education_draft.pptx
Introduction to AI in Higher Education_draft.pptxpboyjonauth
 
“Oh GOSH! Reflecting on Hackteria's Collaborative Practices in a Global Do-It...
“Oh GOSH! Reflecting on Hackteria's Collaborative Practices in a Global Do-It...“Oh GOSH! Reflecting on Hackteria's Collaborative Practices in a Global Do-It...
“Oh GOSH! Reflecting on Hackteria's Collaborative Practices in a Global Do-It...Marc Dusseiller Dusjagr
 
Crayon Activity Handout For the Crayon A
Crayon Activity Handout For the Crayon ACrayon Activity Handout For the Crayon A
Crayon Activity Handout For the Crayon AUnboundStockton
 
18-04-UA_REPORT_MEDIALITERAСY_INDEX-DM_23-1-final-eng.pdf
18-04-UA_REPORT_MEDIALITERAСY_INDEX-DM_23-1-final-eng.pdf18-04-UA_REPORT_MEDIALITERAСY_INDEX-DM_23-1-final-eng.pdf
18-04-UA_REPORT_MEDIALITERAСY_INDEX-DM_23-1-final-eng.pdfssuser54595a
 
How to Configure Email Server in Odoo 17
How to Configure Email Server in Odoo 17How to Configure Email Server in Odoo 17
How to Configure Email Server in Odoo 17Celine George
 
Computed Fields and api Depends in the Odoo 17
Computed Fields and api Depends in the Odoo 17Computed Fields and api Depends in the Odoo 17
Computed Fields and api Depends in the Odoo 17Celine George
 
ECONOMIC CONTEXT - LONG FORM TV DRAMA - PPT
ECONOMIC CONTEXT - LONG FORM TV DRAMA - PPTECONOMIC CONTEXT - LONG FORM TV DRAMA - PPT
ECONOMIC CONTEXT - LONG FORM TV DRAMA - PPTiammrhaywood
 
Presiding Officer Training module 2024 lok sabha elections
Presiding Officer Training module 2024 lok sabha electionsPresiding Officer Training module 2024 lok sabha elections
Presiding Officer Training module 2024 lok sabha electionsanshu789521
 
Hierarchy of management that covers different levels of management
Hierarchy of management that covers different levels of managementHierarchy of management that covers different levels of management
Hierarchy of management that covers different levels of managementmkooblal
 
DATA STRUCTURE AND ALGORITHM for beginners
DATA STRUCTURE AND ALGORITHM for beginnersDATA STRUCTURE AND ALGORITHM for beginners
DATA STRUCTURE AND ALGORITHM for beginnersSabitha Banu
 
Meghan Sutherland In Media Res Media Component
Meghan Sutherland In Media Res Media ComponentMeghan Sutherland In Media Res Media Component
Meghan Sutherland In Media Res Media ComponentInMediaRes1
 
CELL CYCLE Division Science 8 quarter IV.pptx
CELL CYCLE Division Science 8 quarter IV.pptxCELL CYCLE Division Science 8 quarter IV.pptx
CELL CYCLE Division Science 8 quarter IV.pptxJiesonDelaCerna
 
Software Engineering Methodologies (overview)
Software Engineering Methodologies (overview)Software Engineering Methodologies (overview)
Software Engineering Methodologies (overview)eniolaolutunde
 
EPANDING THE CONTENT OF AN OUTLINE using notes.pptx
EPANDING THE CONTENT OF AN OUTLINE using notes.pptxEPANDING THE CONTENT OF AN OUTLINE using notes.pptx
EPANDING THE CONTENT OF AN OUTLINE using notes.pptxRaymartEstabillo3
 
Solving Puzzles Benefits Everyone (English).pptx
Solving Puzzles Benefits Everyone (English).pptxSolving Puzzles Benefits Everyone (English).pptx
Solving Puzzles Benefits Everyone (English).pptxOH TEIK BIN
 

Recently uploaded (20)

Painted Grey Ware.pptx, PGW Culture of India
Painted Grey Ware.pptx, PGW Culture of IndiaPainted Grey Ware.pptx, PGW Culture of India
Painted Grey Ware.pptx, PGW Culture of India
 
Model Call Girl in Tilak Nagar Delhi reach out to us at 🔝9953056974🔝
Model Call Girl in Tilak Nagar Delhi reach out to us at 🔝9953056974🔝Model Call Girl in Tilak Nagar Delhi reach out to us at 🔝9953056974🔝
Model Call Girl in Tilak Nagar Delhi reach out to us at 🔝9953056974🔝
 
Roles & Responsibilities in Pharmacovigilance
Roles & Responsibilities in PharmacovigilanceRoles & Responsibilities in Pharmacovigilance
Roles & Responsibilities in Pharmacovigilance
 
Introduction to AI in Higher Education_draft.pptx
Introduction to AI in Higher Education_draft.pptxIntroduction to AI in Higher Education_draft.pptx
Introduction to AI in Higher Education_draft.pptx
 
“Oh GOSH! Reflecting on Hackteria's Collaborative Practices in a Global Do-It...
“Oh GOSH! Reflecting on Hackteria's Collaborative Practices in a Global Do-It...“Oh GOSH! Reflecting on Hackteria's Collaborative Practices in a Global Do-It...
“Oh GOSH! Reflecting on Hackteria's Collaborative Practices in a Global Do-It...
 
Crayon Activity Handout For the Crayon A
Crayon Activity Handout For the Crayon ACrayon Activity Handout For the Crayon A
Crayon Activity Handout For the Crayon A
 
18-04-UA_REPORT_MEDIALITERAСY_INDEX-DM_23-1-final-eng.pdf
18-04-UA_REPORT_MEDIALITERAСY_INDEX-DM_23-1-final-eng.pdf18-04-UA_REPORT_MEDIALITERAСY_INDEX-DM_23-1-final-eng.pdf
18-04-UA_REPORT_MEDIALITERAСY_INDEX-DM_23-1-final-eng.pdf
 
How to Configure Email Server in Odoo 17
How to Configure Email Server in Odoo 17How to Configure Email Server in Odoo 17
How to Configure Email Server in Odoo 17
 
ESSENTIAL of (CS/IT/IS) class 06 (database)
ESSENTIAL of (CS/IT/IS) class 06 (database)ESSENTIAL of (CS/IT/IS) class 06 (database)
ESSENTIAL of (CS/IT/IS) class 06 (database)
 
9953330565 Low Rate Call Girls In Rohini Delhi NCR
9953330565 Low Rate Call Girls In Rohini  Delhi NCR9953330565 Low Rate Call Girls In Rohini  Delhi NCR
9953330565 Low Rate Call Girls In Rohini Delhi NCR
 
Computed Fields and api Depends in the Odoo 17
Computed Fields and api Depends in the Odoo 17Computed Fields and api Depends in the Odoo 17
Computed Fields and api Depends in the Odoo 17
 
ECONOMIC CONTEXT - LONG FORM TV DRAMA - PPT
ECONOMIC CONTEXT - LONG FORM TV DRAMA - PPTECONOMIC CONTEXT - LONG FORM TV DRAMA - PPT
ECONOMIC CONTEXT - LONG FORM TV DRAMA - PPT
 
Presiding Officer Training module 2024 lok sabha elections
Presiding Officer Training module 2024 lok sabha electionsPresiding Officer Training module 2024 lok sabha elections
Presiding Officer Training module 2024 lok sabha elections
 
Hierarchy of management that covers different levels of management
Hierarchy of management that covers different levels of managementHierarchy of management that covers different levels of management
Hierarchy of management that covers different levels of management
 
DATA STRUCTURE AND ALGORITHM for beginners
DATA STRUCTURE AND ALGORITHM for beginnersDATA STRUCTURE AND ALGORITHM for beginners
DATA STRUCTURE AND ALGORITHM for beginners
 
Meghan Sutherland In Media Res Media Component
Meghan Sutherland In Media Res Media ComponentMeghan Sutherland In Media Res Media Component
Meghan Sutherland In Media Res Media Component
 
CELL CYCLE Division Science 8 quarter IV.pptx
CELL CYCLE Division Science 8 quarter IV.pptxCELL CYCLE Division Science 8 quarter IV.pptx
CELL CYCLE Division Science 8 quarter IV.pptx
 
Software Engineering Methodologies (overview)
Software Engineering Methodologies (overview)Software Engineering Methodologies (overview)
Software Engineering Methodologies (overview)
 
EPANDING THE CONTENT OF AN OUTLINE using notes.pptx
EPANDING THE CONTENT OF AN OUTLINE using notes.pptxEPANDING THE CONTENT OF AN OUTLINE using notes.pptx
EPANDING THE CONTENT OF AN OUTLINE using notes.pptx
 
Solving Puzzles Benefits Everyone (English).pptx
Solving Puzzles Benefits Everyone (English).pptxSolving Puzzles Benefits Everyone (English).pptx
Solving Puzzles Benefits Everyone (English).pptx
 

contraception qs1pdf.pdf

  • 1. 1. A 28-year-old, para 2 had a vaginal delivery 20 days ago. She is exclusively breastfeeding her child. She is sexually active but currently not using a contraceptive method. She is hoping that lactation amenorrhoea method (LAM) will prevent her getting pregnant. 
 Which one of the following will not increase the risk of her getting pregnant? 
 a. Stopping the night feeds
 b. Supplementary feeding
 c. Increase in breastfeeding frequency d. Use of pacifiers
 e. Return of menstruation 2. A 28-year-old woman para 4 seeks advice from GP regarding contraception. She had a vaginal delivery 10 days ago but is happy to use the pills. 
 Which of the following statements regarding initiating contraception is correct during the postpartum period? 
 a. Progesterone-onlypill(POP)canbesafelystartedpriorto21days 
 postpartum. b. Use of COC between 6 weeks and 6 months is recommended in fully 
 breastfeeding women 
 c. Non-breastfeeding women can start COC before day 21 postpartum 
 d. Postpartum women (breastfeeding and non-breastfeeding) can start the 
 POP only after 6 weeks 
 e. Breastfeeding women can start COC in the first 6 weeks postpartum as 
 there is sufficient evidence to prove the safety of COC use while establishing breastfeeding 3. Concerning the use of contraception methods in women with cardiac disease, which one of the following is a correct statement? a. Prophylactic antibiotics are recommended during insertion or removal of 
 intrauterine contraception in women with an increased risk of infective 
 endocarditis b. In women on warfarin therapy the risk of bleeding complication is very 
 high during insertion of a progestogen-only implant and therefore its use should be restricted in these women 

  • 2. c. The intrauterine device should be fitted in the hospital setting if the risk of vasovagal reaction is particularly high d. A causal association has been demonstrated between progestogen-only contraceptive and venous thromboembolism 
 e. The cardiologist should always be involved in deciding whether to use an intrauterine contraceptive device 
 4. A28-year-oldwomanattendsafamilyplanningclinicforcontraceptionadvice. 
 She has tested positive for BRCA1 gene. She has blood pressure (BP) of 140/92 at her last visit with GP. 
 The following are true regarding contraceptive use for her except which one? a. She can be advised that there may be an additional risk of breast cancer with COC use 
 b. She can be advised that there is a reduction in the risk of colorectal cancer with COC use 
 c. In view of her being a BRCA1 gene carrier, her risk of ovarian cancer increases by 30% with COC use 
 d. She can be advised that COC use provides a protective effect against endometrial cancer that continues for 15 years or more after stopping COC 
 e. Hypertension may increase the risk of stroke and myocardial infarction (MI) in those using COC 
 5. A 14-year-old girl was having a suction termination of pregnancy (TOP). She did not want her mother to know. Her mother asks you about why her daughter is in the hospital. 
 The options in the girl’s case include the following except for which one? 
 a. Disclose to the mother
 b. Speak to the patient about disclosure
 c. Encourage patient to disclose to mother d. Advise safe sex practices to patient
 e. Advise reliable contraception to patient 

  • 3. 6. One of the following conditions does not fall into UKMEC (UK Medical Eligibility Criteria for Contraceptive Use) category 1 for using levonorgestrel intrauterine system (LNG-IUS) a. Infections including past pelvic inflammatory disease (PID) with 
 subsequent pregnancy 
 b. Schistosomiasis (with fibrosis of the liver) 
 c. Non-pelvic tuberculosis or malaria 
 d. Infections including past PID without subsequent pregnancy 
 e. Superficial venous thrombosis (varicose veins or superficial 
 thrombophlebitis) 
 7. Which one of the following conditions falls into UKMEC category 3 for using a LNG-IUS?
 a. Complicated valvular and congenital heart disease
 b. Women with known pelvic tuberculosis 
 c. Severe dysmenorrhoea d. Past history of breast cancer with no recurrence in the last 5 years
 e. Undiagnosed breast mass or carriers of gene mutations (e.g. BRCA1) 8. A 26-year-old woman para 1 chooses to have a LNG-IUS as a method of contraception. She has been using other forms of contraception. 
 In which of the following scenarios mentioned below will this woman need the use of extra protection (condoms or abstinence) for 7 days after insertion? 
 a. If the woman is within <12 weeks since last progestogen-only injection b. If the woman is within 3 years of insertion of a subdermal implant 
 c. If the woman is no later than day 1 of the hormone-free interval for pills or patch 
 d. If the woman is within 7 days post-abortion or miscarriage
 e. If the woman is partially breastfeeding, amenorrhoeic and less than 
 3 months’ postpartum 
 9. A 20-year-old woman was brought to A&E with a history of sexual assault
 4 days ago. The perpetrator had forced her to have vaginal intercourse (used condom) and also performed digital anal penetration. You are the doctor in A&E who is now collecting
  • 4. the samples for forensic medical examination. All the following samples are indicated for forensic medical examination except which one? 
 a. Low vaginal swab b. Vulval swab
 c. Peri-anal swab
 d. High vaginal swab e. Anal canal swab 
 10. A 15-year-old girl comes to the family planning clinic for contraception. Her last period was 6 weeks ago and a urine pregnancy test shows a positive result. She expresses a strong wish for termination of pregnancy. 
 The following are basic principles for consenting this young person for the procedure except for which one? a. A patient can only provide consent if she is deemed to have the capacity to 
 consent to treatment. Gillick competence may be applied 
 b. A patient having legal capacity may refuse to have treatment for good/bad 
 reasons or no reason at all and cannot be compelled to have treatment 
 c. Competence/capacity in children below the age of 16 years, for instance, teenage pregnant women; the degree of capacity required varies with the 
 seriousness of the decisions to be taken 
 d. That the young person understands the procedure and has the maturity to 
 understand what is involved 
 e. Consent cannot be revoked by the patient at any time before the treatment 
 is given even if she retains the capacity to do so 11. You are the senior registrar on call for gynaecology. A 15-year-old girl attends emergency department with heavy vaginal bleeding at 16 weeks’ gestation and therefore she is taken to theatre for removal of productions of conception 
 under general anaesthesia. She is haemodynamically unstable and her blood test reveals that the haemoglobin is 5 gm%. The patient is scared and her parents decline to give consent for blood transfusion for their daughter despite explaining that she would die. The parents are Jevohah’s Witness. Her partner is at work. What should the doctor do in this situation? a. Call the trust solicitor for advice
 b. Call the senior sister in charge to talk to the parents
  • 5. c. Give blood transfusion to save the girl
 d. Listen to the parents
 e. Wait for her partner to come For the following scenarios choose the most appropriate contraceptive method from the list above A. Copper bearing intrauterine device B. Levonorgestrelintrauterinesystem C. Norethisterone5mg D. Desogestrel 75 mcg E. IMmedroxyprogesteroneacetate150mg F. Levonorgestrel 150 mcg and ethinylestradiol 30 mcg G. Desogestrel 150 mcg and ethinylestradiol 30 mcg H. Cyproterone acetate 2mg and 35 mcg ethinylestradiol I. Endometrial ablation J. Hysterectomy 12) A 43 year old requests contraception. She has 3 children with her ex-husband. She has been in a new relationship for the past 3 months and has been using condoms. She has no personal or family history of venous thromboembolism, cancer or heart disease. Her periods are regular occurring every 28-30 days. She advises you these are very heavy and reports she has tried taking tablets for these in the past but they didn't help. She has no plans for anymore children. She is a non-smoker. On examination BP 132/86, HR 84, BMI 30.5 kg/m2 13) The gastroenterology team ask your advice regarding contraception for a 40 year old alcoholic who has been admitted with decompensated liver cirrhosis. She is having regular unprotected sexual intercourse. She smokes 20 cigarettes per day. Her BP is 102/60, HR 100 reg, BMI 19.5 kg/m2far). 14) A 39 year old requests contraception. She has been in a new relationship for 3 months. She has no personal or family history of VTE, cancer or heart disease. Periods are regular every 28-30 days. She reports these are very heavy, mefanamic acid tablets in the past didn't help. She has no plans for children in the next few years at least and isn't sure if she wants to have children at all. She is a non-smoker. Obs: BP 136/73, HR 74, BMI 33.5 kg/m2. Pelvic ultrasound shows 3 small fibroids measuring 1.0 cm - 1.5 cm and a bicornuate uterus 15) A 22 year-old woman requests contraception. She has tried Microgynon in the past but this didn't help her acne which bothers her. She has no personal or family history of venous thromboembolism, cancer or heart disease. Her periods are regular occurring every 28-30 days. She
  • 6. is a non-smoker. On examination BP 118/68, HR 82, BMI 27.5 kg/m2 and you note moderate facial acne with some dark hair visible above the upper lip 16) A 24 year old woman requests contraception. She has used condoms in the past but has started a new relationship and and does not want to use barrier methods. Her cycles are regular every 30 days and she is a non-smoker. On examination BP 118/68, HR 82 and BMI 27.5 kg/m2 . You note she a diagnosis of systemic lupus erythematous with the last flare 18 months ago. 17) A 42 year old requests contraception. She has recently started a new relationship and has been using condoms. She has no personal or family history of venous thromboembolism, cancer or heart disease. Her periods are regular occurring every 28-30 days. She is a non-smoker. On examination BP 128/78, HR 76, BMI 30.5 kg/m2 18) A 16 year old patient is seen in family planning clinic as she has been prescribed antibiotics for a chest infection by her GP. She didn't reveal to her GP that she was taking the combined oral contraceptive pill as the GP is a family friend and she didn't want the GP to know she was sexually active. One of her friends has scared her by telling her antibiotics stop contraception from working. Which of the following antibiotics requires additional contraceptive precautions? A) Amoxicillin
 B) Clarithromycin
 C) Doxycycline
 D) Ciprofloxacin
 E) None of the above 19) A patient wants to use her diaphragm for contraception following delivery of her child. She is not breastfeeding. When would you advise the patient it is safe to start using this? A) Can use immediately B) 3 months post delivery C) 3 weeks post delivery D) 4 weeks post delivery E) 6 weeks post delivery 20) A 23 year old woman asks for your advice regarding contraception. She has tried multiple combined and progesterone only oral contraceptives but had to discontinue due to mood swings or heavy bleeding. She doesn't want long acting reversible contraception and doesn't like the feel of condoms. She is considering the use of a diaphragm with spermicide. What would you advise her regarding timing of diaphragm removal after sexual intercourse?
  • 7. A) Remove immediately after intercourse B) Remove 12 hours after intercourse C) Remove 6 hours after intercourse E) Remove 1 hour after intercourse 
 21) A 22 year old woman asks for your advice regarding contraception. She has tried multiple combined and progesterone only oral contraceptives but had to discontinue due to mood swings or heavy bleeding. She doesn't want long acting reversible contraception and doesn't like the feel of condoms. She is considering the use of a diaphragm with spermicide. What would you advise her regarding timing of spermicide application prior to sexual intercourse? A) Spermicide should be reapplied if diaphragm or cap has been in situ for 3 hours or more and sex is to take place B) Spermicide should be reapplied if diaphragm or cap has been in situ for 2 hours or more and sex is to take place C) Spermicide should be reapplied if diaphragm or cap has been in situ for 1 hour or more and sex is to take place D) Spermicide should be reapplied if diaphragm or cap has been in situ for 6 hours or more and sex is to take place E) Spermicide should be reapplied if diaphragm or cap has been in situ for 30 minutes or more and sex is to take place 22) A patient wants to restart her combined contraceptive pill following the delivery of her child. She is not breastfeeding. When would you advise the patient to start taking their contraceptive pill? A) Start immediately B) 2 weeks post delivery C) 3 weeks post delivery
  • 8. D) 4 weeks post delivery E) 6 weeks post delivery 23) A 24 year old patient is considering starting combined hormonal contraception (CHC). She is worried about the possibly of thromboembolism having read about some pills being banned in France. Which of the following has the lowest risk of venous thromboembolism? A) Ethinylestradiol plus cyproterone acetate B) Ethinylestradiol plus Norethisterone C) Ethinylestradiol plus Drospirenone D) Ethinylestradiol plus Desogestrel E) Ethinylestradiol plus Gestodene 24) A 22 year old woman asks for your advice regarding contraception. She has tried multiple combined and progesterone only oral contraceptives but had to discontinue due to mood swings or heavy bleeding. She doesn't want long acting reversible contraception. She asks about male condoms. Assuming correct use what would you advise the patient regarding effectiveness of the male condoms in preventing pregnancy? A) 88% B) 92% C) 95% D) 98% E) 100%
  • 9. 25) Which of the following progestogens is associated with the highest risk of venous thromboembolism when used with Ethinylestradiol in combine hormonal contraception? A) Drospirenone B) Levonorgestrel C) Norgestimate D) Etonogestrel E) Norethisterone 26) A patient wants to restart her progesterone only pill following the delivery of her child. She is not breastfeeding. When would you advise the patient to start taking their contraceptive pill? A) Advise pill contraindicated B) Can start immediately C) 4 weeks post delivery D) 5 weeks post delivery E) 6 weeks post delivery 27) A 24 year old woman has missed her last 2 combined oral contraceptive pills and had UPSI 24 hours ago. The last pill she took was day 3 of her new pack. What would you advise? A) Advise her to take both missed pills now then resume normal pill taking. B) Advise her to take missed pills now, resume normal pill taking with next pill but to run back to back packs i.e. no pill free break
  • 10. C) Advise her to take 1 pill now and next pill when due. D) Advise her to take next pill at normal time and resume pill taking. E) Emergency contraception 28) A patient wants to ask your advice about contraception following delivery of her baby. She is not breastfeeding and would like the Mirena coil. When would you advise the patient it would be safe to have this fitted? A) Anytime B) 3 weeks after delivery C) 4 weeks after delivery D) 6 weeks after delivery E) 3 months after delivery 29) A 26-year-old woman comes to your clinic requesting emergency contraception. She is not currently in a stable relationship. She had unprotected intercourse 4 days ago. She is a para 1+2. Her son is 7 months old and she has had two terminations of pregnancy in the past. She is on day 20 of her cycle and has taken EllaOne® 10 days ago. She has no significant medical or surgical history. What option of emergency contraception is best suited for her? a) Copper IUCD insertion and STI risk assessment b) Intrauterine system insertion c) Norethisterone tablet d) Repeat dose of ellaOne® e) Two tablets of Levonelle®
  • 11. 30) A 15-year-old girl is 10 weeks pregnant and has self-referred requesting termination of pregnancy. What is not part of a routine consultation? a) Check for Gillick’s competency as she is less than 16 years of age b) Check Rhesus status as she may need anti-D c) Consider child safeguarding issues by asking about details of partner d) Explain risks and side effects of procedure e) Inform parents as would need general anaesthesia for procedure 31) A 35-year-old para3 comes to clinic for contraceptive advice. She was advised to come off the combined pill as she developed migraines with aura while taking it. She is now using the diaphragm but is looking for something more reliable. With regards to duration of action of long-acting reversible contraception, which one of these statements is not correct? a) Depo-Provera® injection is effective for 13 weeks b) Mirena coil is effective for 5 years c) Multiload copper coil is effective for 5 years d) Nexplanon® is effective for 5 years e) T-Safe® Cu380A is effective for 10 years 32) A 27-year-old nulliparous woman comes to the outpatient clinic requesting Depo- Provera® injection. She wants to know about the non-contraceptive benefits and risks with injectable progesterones. Which of the following is incorrect? a) Anaemic women may benefit from using injectable progestogens b) Depot medroxy progesterone acetate is associated with increase in bodyweight c) Injectable contraceptives have been associated with an increased risk of developing venous thromboembolism in women with a previous history of venous thromboembolism after immobilisation d) It can reduce pain due to endometriosis e) Women with sickle cell disease may notice a decrease in pain during sickle cell crisis
  • 12. 33) You are going to prescribe combined transdermal patches for a 15 year old girl with cerebral palsy as she is unable to cope with her periods. For the combined transdermal patches to achieve the same effect as continuous oral contraceptive pill, for how long the patches should be applied serially for? a) 3 weeks followed by 7-day withdrawal period b) 4 weeks followed by 7-day withdrawal period c) 5 weeks followed by 7-day withdrawal period d) 7 weeks followed by 7-day withdrawal period e) 9 weeks followed by 7-day withdrawal period 34) A 30-year-old woman had a copper intrauterine device inserted immediately after she had a normal vaginal delivery. She presents 3 weeks later because she cannot feel the thread. What would be the most appropriate step to take in her management? a) Perform a pregnancy test and then arrange an ultrasound scan b) Perform a pelvic examination first, then a pregnancy test, and then an ultrasound scan if the test is negative c) Perform an ultrasound scan and if the device is not seen, request a urine pregnancy test d) Perform an ultrasound scan and if the device is not located, perform a serum hCG assay e) Perform a pelvic examination and if the thread cannot be seen, perform an ultrasound scan and if negative, perform a pregnancy test 35) In which group of women with a heart disease undergoing an IUD insertion does the FSRH recommend the administration of prophylactic antibiotics? a) Those with a history of rheumatic fever b) Those with a prosthetic valve c) Those with congenital heart disease d) Those with hypertrophic cardiomyopathy e) Those with septal defects
  • 13. 36) A 30-year-old attends for contraceptive advice. She suffers from pulmonary hypertension and does not wants to get pregnant. What WHOMEC (UKMEC) category will you place this patient in with respect to offering her contraception? a) UKMEC category I b) UKMEC category II c) UKMEC category III d) UKMEC category IV e) UKMEC category V 37) Which progestogen in combined hormonal contraception is associated with the highest risk of venous thromboembolism? a) Gestodene b) Levonorgestrel c) Medroxyprogesterone d) Norethisterone e) Norgestimate 38) A 45-year old is on the progestogen-only pill containing desogestrel. What would be considered a missed pill in this woman? a) Delay in taking the pill by more than 3 hours b) Delay in taking the pill by more than 6 hours c) Delay in taking the pill by more than 12 hours d) Delay in taking the pill by more than 8 hours e) Delay in taking the pill by more than 24 hours 39) When counselling a man regarding vasectomy, which of the following is it important to stress a) Contraception should be used for a further 3 months with a negative semen analysis at the end of that time b) Reversal is readily available as an NHS procedure c) The procedure is irreversible d) There is a small increased risk of testicular cancer following the procedure
  • 14. e) Vasectomy has a higher failure rate than female sterilization 40) Which of the following statements is the most appropriate concerning long-acting reversible contraception and intrauterine devices (IUDs) a) Gynefix is currently the only frameless IUD licensed in the UK. b) IUDs available in the UK are licensed for a variable time period from 5 to 8 years. c) The most effective IUDs contain at least 280 mm2 of copper d) The risk of ectopic pregnancy increases in women using IUDs e) Unbanded devices are the most effective IUD presently available 41) Concerning LARC injectables and implants, below are true EXCEPT: a) The major mechanism of action of depomedroxyprogesterone acetate (DMPA) is to thicken cervical mucus. b) Progestogen implants are associated with irregular vaginal bleeding. c) The failure rate of progestogen implants is higher in women with a BMI 40. d) DMPA does adversely affect bone density in long-term use and, therefore, it is not recommended as a first-line contraceptive in young women. e) Approximately 50% of women in the UK discontinue DMPA within 1 year 42) Concerning progestogen-only pills (POPs), which of the following is true? a) Additional contraceptive precautions should be taken during antibiotic treatment b) In women undergoing treatment for breast cancer, POPs increase the rate of disease recurrence c) They are associated with an increased risk of thromboembolic disease d) They are contraindicated in women with insulin-dependent diabetes e) They should be discontinued 4 weeks prior to major surgery 43) When counselling a woman for postcoital contraception (PCC), what should you advise a) Intrauterine PCC can be inserted up to Day 19 of a 28-day cycle. b) Mifepristone can be used for PCC c) STI screening results should be available prior to fitting intrauterine PCC d) The intrauterine system is licensed for PCC e) Ulipristal acetate is as effective as an IUD 44) An 18-year-old p1+0 (1-year-old child, normal vaginal delivery) attends her local sexual reproductive health clinic requesting contraception. Her first pregnancy was unplanned and she wants a reliable method of contraception, at the same time planning for a further pregnancy in 12–18 months. She is in a steady relationship and has been with the same partner for 2 years. Her periods are irregular, heavy and painful, and have got worse over recent months, with a cycle of 5–7/21–38 days. She has also noted increased facial hair, requiring electrolysis.
  • 15. She has previously been diagnosed with PCOS. She has no relevant family history. On examination she looks well: • height: 153 cm • weight: 94 kg • BP: 120/75 mmHg. A: Condoms B: Levonorgestrel IUS (Mirena®) C: Depo medroxyprogesterone acetate (Depo-Provera®) D: Noriday® progestogen-only pill E: Dianette® combined oral contraceptive pill Choose the method of contraception that is the most suitable for her 45) Miss Y is a 22-year-old nulliparous young woman who presents at her local sexual reproductive health clinic having had unprotected sexual intercourse on day 16 of a regular 28-day cycle. She is in a new relationship, having had intercourse last night with her 27- year-old boyfriend who she met last week at a party. She has not had any other episodes of unprotected intercourse this cycle. She suffers from epilepsy and takes carbamazepine 400 mg twice daily. She is otherwise well with no relevant family history. She looks well and her measurements are as follows: • height: 159 cm • weight: 58 kg • BP: 115/65 mmHg. A: Insert a copper IUD as postcoital contraception and continue to use the IUD as her preferred method of contraception B: Levonelle 1.5 mg immediately as postcoital contraception, repeated after 12 hours. Condoms and commence Depo-Provera® to be administered every 12 weeks at the onset of her next period C: Levonelle 1.5 mg immediately as postcoital contraception, repeated after 12 hours. Condoms and commence Cerazette® progestogen-only pill at the onset of her next period D: Levonelle 1.5 mg immediately as postcoital contraception. Condoms and commence Depo-Provera® to be administered every 10 weeks at the onset of her next period E: Levonelle 1.5 mg immediately as postcoital contraception. Condoms and commence Microgynon 30 combined oral contraceptive at the onset of her next period Choose the method of contraception that is the most suitable for her 46) Mrs P, a 32-year-old para 4+0, has just delivered a baby boy (SVD) following an unplanned pregnancy, having recently separated from her husband and entered into a new short-term relationship. She is a week postpartum and requesting a reliable form of contraception. She has a family history of VTE in a first-degree relative and is known to carry a protein S deficiency but has never had a VTE episode herself. She is bottle feeding, her puerperium has been uncomplicated and she is keen to start contraception as soon as possible. • Height: 160 cm • Weight: 65 kg • BP: 135/80.
  • 16. A: Advise regarding STI and condom use, follow-up appointment to discuss male or female sterilisation B: Immediate insertion of copper IUD (T-Safe® Cu380A), advise regarding STI and condom use C: Commence progestogen-only pill (Cerazette®) immediately, advise regarding STI and condom use D: Commence combined oral contraceptive (Microgynon® 30) 4 weeks postpartum, advise regarding STIs and condom use E: Advise regarding STI and condom use, and injection of Depo-Provera® after 6 weeks Choose the method of contraception that is the most suitable for her 47) A 35-year-old para 1+1 has recently undergone surgical management of miscarriage following a proven molar pregnancy. The hCG levels are being monitored and are still significantly elevated. She is continuing to experience irregular vaginal bleeding following the evacuation of the uterus. On examination: • height: 156 cm • weight: 58 kg • blood pressure: 110/65 mmHg She is in a long-term relationship and is requesting contraception. A: Combined hormonal pill containing 30 microgram ethinylestradiol and a first-generation progestogen B: Levonorgestrel IUS (Mirena®) C: Depo-medroxyprogesterone acetate (Depo-Provera®) D: The desogestrel progestogen-only pill E: Subdermal implant Which method of contraception is contraindicated? 48) A 26-year-old woman comes to your clinic requesting emergency contraception. She is not currently in a stable relationship. She had unprotected intercourse 4 days ago. She is a para 1+2. Her son is 7 months old and she has had two terminations of pregnancyin the past. She is on day 20 of her cycle and has taken EllaOne® 10 days ago. She has no significant medical or surgical history. What option of emergency contraception is best suited for her? a) Copper IUCD insertion and STI risk assessment b) Intrauterine system insertion c) Norethisterone tablet d) Repeat dose of ellaOne® e) Two tablets of Levonelle®
  • 17. 49) A 35-year-old para3 comes to clinic for contraceptive advice. She was advised to come off the combined pill as she developed migraines with aura while taking it. She is now using the diaphragm but is looking for something more reliable. With regards to duration of action of long-acting reversible contraception, which one of these statements is not correct? a) Depo-Provera® injection is effective for 13 weeks b) Mirena coil is effective for 5 years c) Multiload copper coil is effective for 5 years d) Nexplanon® is effective for 4 years e) T-Safe® Cu380A is effective for 10 years . 50) A 26-year-old para 1 comes to your clinic requesting contraception. She mentions that her maternal aunt had deep vein thrombosis (DVT) and is on warfarin. She is worried that if she takes hormonal contraception she will also develop DVT. Which of the following advice regardling DVT is incorrect. a) Combined oral contraceptives containing a third generation progestogen is associated with an increased risk of venous thromboemolism (VTE) when compared with those containing second generation progestogens b) The risk of VTE increases with the number of years of usage of a combined hormonal pill c) There is little or no data to suggest that there is an increased risk of VTE with oral or injectable progesterone only contraceptives methods d) Thrombophillia screen is not recommended routinely before use of combined hormonal contraceptive e) Observational studies have reported that the transdermal patch has a somewhat similar level of VTE risk to combined oral contraceptives containing second-generation progestogens 51) A couple have come to your clinic requesting sterilisation. After going through the risks and benefits of the procedures they opt for vasectomy. Which of the following should not be taken into consideration during vasectomy? a) 12-week postvasectomy is considered an optimal timing for scheduling first postvasectomy semen analysis b) Clips are not recommended for occlusion of the vas deferens as it is associated with a higher failure rate c) If motile sperms are found in a fresh semen sample 3 months postprocedure, the vasectomy is be considered a failure d) Routine histological examination of the vas deferens is no longer routinely recommended e) Warming local anaesthetic to body temperature before infiltration reduces pain due to injection
  • 18. 52) You are a ST5 and are currently in a minimal access surgery placement. Your consultant is training you in performing hysteroscopic sterilisation on a 38-year-old woman. She has had three previous normal vaginal deliveries and a history of Chlamydia trachomatis infection at age 19; otherwise no other history of note. At the end of the procedure you are asked to arrange a follow-up ultrasound scan in 3 months' time in order to confirm occlusion of the fallopian tubes. Where in the fallopian tubes would the micro-inserts be placed? a) Ampullar b) Infundibulum c) Intraluminal d) Intramyometrial e) Isthmus 53) Which of the following is correct concerning Nonoxynol-9 a) Regular use decreases risk of sexually transmitted HPV that can cause cervical cancer b) Regular use protects against sexually transmitted infections such as gonorrhoea and chlamydia c) Regular use reduces the risk of HIV transmission d) Condoms lubricated with Nonoxynol - have a longer shelf life than those without e) Is a type of organic surfactant and an active constituent of spermicide 54) A 47 year old woman seeks advice about continuing the combined oral contraceptive pill ( COCP ) . She is normotensive and a non smoker with a BMI of 25. She has no other medical history and no significant family history. She is concerned that the COCP may give her additional health risks. Which of the following malignancies would you advice she may have a small additional risk of developing due to taking the COCP a) breast cancer b) colorectal cancer c) endometrial cancer d) lung cancer e) ovarian cancer 55) A 26 year old woman presents to the emergency gynaecology clinic requesting emergency contraception ( EC) . She had unprotected sex 6 days ago. She is not currently using any contraception, having not had a partner for a year. She has a regular 28 day menstrual cycle, which can be heavy. The first day her last period was 15 days ago.What emergency contraception option, if any, would you advise? a) A copper bearing intrauterine device b) A MIRENA coil c) It is too late for EC d) Levonelle e) Ulipristal acetate
  • 19. 56) A 25 year old woman with a bicornuate uterus attends the emergency gynaecology unit requesting EC. She has been on holiday and forgot to take her contraceptive pill for 3 days in the first week of the calendar pack and had unprotected sexual intercourse ( UPSI ) four days ago. She is in good health. Which of the following is the recommended EC? a) Copper IUCD b) MIRENA IUS c) Levonorgestrel d) Mifepristone e) Ulipristal acetate 57) What is the risk of VTE in women using combined hormonal contraception? a) 1 per 10,000 b) 2-4 per 10,000 c) 5-12 per 10,000 d) 15 - 20 per 10,000 e) 25 - 30 per 10,000 58) When is the risk of VTE highest after starting combined hormonal contraception? a) In the first one month b) In the first 4 month c) In the first year d) In the first 3 years e) Remains the same while used 59) A 34 year old woman has an elective caesarean section because she is HIV positive She is taking anti-retroviral drugs and wishes to use depo-medroxyprogesterone acetate for contraception. Depo - medroxyprogesterone acetate can be used without restriction ( UKMEC 1) in women taking which anti- retroviral drugs? a) Indanivir b) Nevirapine c) Lamivudine d) Zidovudine + Efavirenz e) All the drugs listed 60) A 34 year old woman attends the emergency gynaecology clinic requesting emergency contraception. her LMP was 2 weeks ago and she has a regular 28 day cycle. She had unprotected intercourse 24 H earlier. She suffers from epilepsy and is treated with phenytoin. Which one is the most appropriate option?
  • 20. a) Levonorgestrel 1500mcg x 2 doses 12 hours apart b) Levonorgestrel 750mcg x 2 doses 12 hours apart c) Ethinyl-oetradiol 500mcg d) Levonorgestrel 1500mcg followed by 750mcg after 12 hours e) Ethinyl-oestradiol 100mcg + levonorgestrel 750mcg 61) A 23 years old nulliparous woman attends AE. She gives a history of UPSI the previous night. Her periods are regular 3-4/30. She had sexual intercourse with a stranger. Her last menstrual period was 2 weeks ago. What is the most appropriate step? a) Levonorgestrel 2 tablets stat and UPT in 2 weeks time if there has been no period b) Reassurance since UPT is negative and to cont male condom c) Levonorgestrel 2 tablets stat, UPT in 2 weeks time if there has been no period and offer STI screening d) Copper IUCD stat and UPT in 2 weeks time if there has been no period e) Levonorgestrel - releasing intrauterine system as benefit for long term contraception 62) A 17 year old girls requests EC. She gives a history of unprotected sexual intercourse 40 hours ago. She is unsure of when her LMP was as her period are irregular. Her regular contraceptive method is the male condom. What is the next management ? a) Sustain further history about the partner ( age, relationship, promiscuity) b) Offer screening for STD c) Assess Gillick/Fraser competency d) Azithromycin stat dose e) Pregnancy test now and may need to be repeated later 63) A 25 year old woman requests emergency contraception following UPSI on one occasion 4 days ago. She is currently taking a proton pump inhibitor for gastro- oesophageal reflux. What EC would you recommend? a) Copper IUCD b) Levonorgestrel c) MIRENA d) Mifepristone e) Ulipristal acetate 64) The United Kingdom Medical Eligibility Criteria ( UKMEC) offers guidance to clinicians when considering the use of contraceptives. There are 44 categories, with category 1 being generally suitable and Category 4 completely contraindicated. when considering the combined oral contraceptive pill, which of the following factors would be considered as a Category 4( completely contraindication )? a) Age 30 and smoking ( more than 35/day)
  • 21. b) Family history of breast cancer with a known BRCA gene mutation c) History of DVT d) Hypertension 140/90 e) Three weeks postnatal and breastfeeding 65) A 34 year old woman with BMI of 34 presents for contraceptive advice. She has had treatment for DVT. Concerning hormonal contraception and thromboembolic disease, which of the following is true? a) For women who are post part and not breast feeding , combined hormonal contraception can be started immediately b) for women with current VTE on anticoagulants, the POPs is contraindicated c) Hormonal contraception can be started immediately after a 1st and 2nd trimester TOP d) POPs, implants and MIRENA have an increased risk of VTE e) Women with factor V leiden have up to a five-fold increase in the risk of VTE with COCP use 66) A 24 year old woman attends your clinic for EC after 2 episodes of UPSI over the last 24 hours. She is currently on treatment for tuberculosis. Which is the most appropriate contraceptive to prescribe her? a) Copper IUD b) Implanon c) Levonorgestrel d) MIRENA e) Ulipristal acetate 67) A 33 yo woman consults you for contraception following her second delivery. She does not want oral medication and asks about the combined vaginal ring. Which of the following is correct? a) it contains ethynylestradiol and levonorgestrel b) it is changed every 6 months c) it is uneffected by enzyme inducing drugs d) It should be kept refrigerated before use e) The risk of VTE is reduced when compared with COC 68) A 26 year old woman with known case of heavy menstrual bleeding would like to try on MIRENA coil . She also keen for LARC. What is common risk associated with the MIRENA coil ? a) Expulsion, occuring in 1% of cases b) Expulsion , occuring in 5% of cases c) Failed contraception, occurring in 1% of cases d) Perforation, occuring in 1% of cases e) Perforation, occuring in 5% of cases.
  • 22. 69) You review a 17 - year old woman who requests emergency contraception after four episodes of unprotected sexual intercourse over the last five days. She is asthmatic, with an admission to ITU last year. Which method of contraception is the most appropriate to prescribe? a) Copper intrauterine device b) Levonorgestrel pill c) Mirena coil d) Progesterone - only pill e) Ulipristal acetate 70) A 32-year-old woman comes to see you following her second pregnancy, which resulted following contraceptive failure. All the following contraceptive methods primarily work by inhibiting ovulation, except a) Combined hormonal contraceptive pill (CHC) b) Combined vaginal ring ( CVR) c) Depot medroxyprogesterone acetate ( DMPA) d) Levonorgestrel-containing intrauterine system ( IUS) e) Subdermal contraceptive implant 71) You discuss ulipristal acetate ( UPA) - containing emergency contraception for a 24- year-old woman requesting emergency contraception. Which of the following is correct? a) A double dose should be given if the woman is using liver enzyme-inducing medication b) It can be administered beyond 120 hours c) It is considered first-line for women taking antacids or H2 receptor antagonists d) It should only be used once in a menstrual cycle e) The primary mechanism of action is prevention of implantation 72) A 40 year- old on the COCP wishes to discuss the benefits of continuing with the pill with regards to ovarian cancer protection. What advice will you offer her? a) COCP reduce the risk of ovarian cancer by approximately 25% b) Low dose COCP ( 35ug ethinyl estradiol) do not confer adequate protection compared to high dose COCP c) The decrease risk for ovarian cancer from COCP use is independent of use after the first 12 months d) The protective effect of COCP on ovarian cancer risk ceases 5-10 years after stopping the pills e) Women who have BRCA 1 and BRCA 2 mutations should not go on the COCP
  • 23. 73) A 33 year- old healthy lady , para 2 missed to take 3 pills of her COC ( pills day 5 - day 7). She had a sexual intercourse 48 hours ago. a) Emergency contraception b) Take missed pills as soon as possible, continue remaining pills and emergency contraception c) Take her 2 missed pills as soon as possible, continue remaining pills and emergency contraception d) Take recent missed pills as soon as possible, continue remaining pills, emergency contraception and additional contraceptive for 7 days e) Take recent missed pills as soon as possible, continue remaining pills, no need emergency contraception but needs additional contraceptive for 7 days 74) 25 year-old healthy lady, Para 1 missed to take 4 pills of her COCP on day 18 - day 21. She presented to you after she had sexual intercourse 48 hours ago. a) Take recent missed pills as soon as possible, continue remaining pills, emergency contraception and additional contaceptive for 7 days b) Omit pill free interval and start new pack the next day c) No additional contraception needed d) Emergency contraception and additional contraceptive for 7 days e) UPT if no menses in next cycle 75) A 40 years old woman requested for contraception. She is keen for CHC. Disadvantages of CHC explained to her. Which of the following is not true? a. Increase risk of breast cancer b. Increased risk of colorectal carcinoma c. Reduced risk of benign breast disease d. Increased risk of cervical cancer e. Increased risk of ischeamic stroke
  • 24. EMQ The following clinical scenarios describe situations in which the use of one or more contraceptive methods is not advisable. For each scenario, decide which contraceptive method or methods should be avoided from the list of available options; assume that there are no other factors that influence contraceptive suitability. Each option may be used once, more than once, or not at all 1. A 30- year old nulliparous woman with systemic lupus erythematous ( SLE ) with positive or unknown antiphospholipid antibodies 2. A 26-year-old woman with chronic viral hepatitis with normal liver function. 3. A 27-year-old nulliparous woman with a history of an ectopic pregnancy managed by salpingectomy. A All hormonal contraceptive methods, excluding the levonorgestrel-containing intrauterine system ( IUS) B All hormonal contraceptive methods, including the levonorgestrel-containing intrauterine system ( IUS) C Combined hormonal contraception ( CHC) , combined vaginal ring ( CVR) , combined transdermal patch ( CTP) D Combined hormonal contraception ( CHC) and progestogen-only injectables ( depo-provera) E Copper-containing intrauterine contraceptive device ( IUD ) F Copper-containing intrauterine contraceptive device (IUD) and progestogen-only pill ( POP) G Levonorgestrel-containing intrauterine system ( IUS) H Levonorgestrel-containing intrauterine system ( IUS) and copper-containing intrauterine device ( IUD ) I Progestogen-only implants J Progestogen-only injectables ( depo-provera) K Progestogen-only pill ( POP) L Progestogen-only pill ( POP) and progestogen-only implants M All of the above N None of the above
  • 25. Match the most appropriate management to each scenario: 4. A 43-year-old woman started a combined oral contraceptive pill three months ago. She has started having irregular bleeding. She is known to have PCOS ( polycystic ovarian syndrome). She is a non-smoker and is up to date with her smears. 5. A 24- year-old woman presents with irregular bleeding six months after starting the combined oral contraceptive pill. She has no medical problems. She has recently started a new relationship. She is up to date with her smears. 6. A 50-year-old woman has used the levonorgestrel IUS ( intrauterine system) for heavy menstrual bleeding for the past five years and is amenorrhoiec. She now wonders whether she has gone through the menopause, and therefore can have the coil removed. 7. A 52-year-old woman had a copper coil fitted at the age of 43. Her last menstrual period was 13 months ago. She wishes to know whether she has gone through the menopause and therefore can have the coil removed. A Leave in for further 12 months and then remove or check two FSH levels, six weeks apart, and if both over 30 IU/L, remove B Take a full sexual history and commence a 30 mug or 35 mug combined oral contraceptive pill. If not resolving, consider endometrial biopsy C Take a full sexual history and investigate for sexually transmitted infections if appropriate D Take a full sexual history and cervical smear E Stop the contraceptive and advise trying a long-acting reversible contraceptive such as the implant F Take a full sexual history, do a pelvic examination and cervical smear. If normal, consider adding mefenamic acid alongside G Remove the device now H Leave in for a further 24 months and then remove or check two FSH levels six weeks apart and if both over 30 IU/L , remove after 12 months A Natural family planning B Male condoms C Female condoms D Diaphragm E Combined oral contraceptive pill F Evra G Nuva Ring
  • 26. Listed are recognised methods of contraception. From this list, select the single contraception method that is clinically most appropriate given the clinical scenario described. 10. A 30-year-old woman who continuously forgets to take contraception. She wishes long intervals between needing to renew her contraception. She is nulliparous in a stable relationship. She is not keen for anything to be inserted into her uterus. 11. A 40-year-old smoker with a BMI of 40 and has 3 previous caesarean sections. She feels she has completed her family and wants permanent contraception. 12. An 18-year-old woman with painful periods which are irregular. She is keen on contraception, preferably something that will regulate her menses. She is nulliparous. She is not keen to take pills and is scared of needles. She does not want any invasive form of contraception. 13. A 48-year-old woman with irregular cycle heavy menstrual bleeding. She is fed up with using POP for contraception. She is in a stable relationship and her family is complete. She is considering HRT when menopause occurs. 14. A 24-year-old with strong family history of ovarian cancer. Has had one child and is planning to have bilateral oopherectoy following birth of second child. She seeks effective contraception that would be beneficial to her long term health. 15. A 25-year-old Para 1 , known case of Rheumatoid arthritis in remission, presented to your clinic to discuss reliable long acting reversible contraception. H progestogen only pill I MIRENA J Copper IUD K Depot medroxyprogesterone acetate L Implanon M Hysteroscopic sterilisation N Laparoscopic sterilisation A Natural family planning
  • 27. Match the most appropriate advice to each scenario. 16. A 28-year-old woman misses two consecutive 30mcg combined pills in the second week of taking her packet ( days 9 and 10) . She has been taking all her pills on days 1-7. She had sexual intercourse with her boyfriend on day 8 . 17. A 30-year- old Para 3, missed to take 1 pill of her COC on pills day 6. She presented to you after she had sexual intercourse 48 hours ago. A Take both missed pills, take the remaining pills as usual, condoms should be used for the next seven days or sexual intercourse avoided in case further pills are missed. No indication for emergency contraception B Take the most recent missed pill, take the remaining pills as usual, condoms should be used for the next seven days or sexual intercourse avoided in case further pills are missed, No indication for emergency contraception C Take both missed pills, take the remaining pills as usual , condoms should be used for the next seven days or sexual intercourse avoided in case further pills are missed. Emergency contraception should also be advised. D Take the most recent missed pill, take the remaining pills as usual, condoms should be used for the next seven days or sexual intercourse avoided in case further pills are missed, Emergency contraception should also be advised E Omit pill free interval and start new pack the next day F Administer the next injection, offer emergency contraception, additional contraception or abstinence should be advised for the next seven days and a pregnancy test should be carried out in 21 days G The next injection should not be administered, emergency contraception should be offered and a pregnancy test should be carried out in 21 days H Administer the next injection, emergency contraception not required, additional contraception or abstinence for the next seven days I Prescribe additional contraception and start new ring immediately without a ring free interval J Start new ring immediately without a ring free interval K Remove ring and wait for withdrawal bleed, if no menses for UPT testing L Apply new patch now and delay change by 48 hours M Apply new patch now but keep the same date of change N Apply new patch and use barrier contraception for 7 days O Take missed pills as soon as possible, take next pill at usual time, additional contraceptive for 2 days, offer emergency contraception P Reassure
  • 28. 18. A 36-year-old woman phones for advice because she has missed her contraceptive pill. Her LMP was 12 days ago and she has regular 28 day cycle. She is using a standard progestogen-only pill and has missed her one pill by 8 hours. She had unprotected intercourse 6 hours earlier. 19. A 36-year-old woman phones for advice because she has missed her contraceptive pill. Her LMP was 12 days ago and she has a regular 28 day cycle. She is using Cerazette and has missed her one pill by 8 hours. She had unprotected intercourse 6 hours earlier. 20. A 32-year -old woman had her last DMPA ( depo-provera) injection 14 weeks and two days ago. She had sexual intercourse with her partner two days ago. 21. A 34-year-old Para 3 used combined vaginal ring as her contraception of choice, she already wearing the ring for 32 days and planned to remove it. Best advice is; 22. A 36-year-old woman on day 8 of her regular menstrual cycle has not changed the patch for 48 hours. 23. A 25-year-old healthy woman day 30 post partum currently started to have her menses back and she had take COCP at day 4 of her menses. 24. A 30-year-old lady now day 40 post partum and has her period of amenorrhoea since delivery took estradiol valerate as her contraceptive method at day 40 post partum. A Start at Day 1 menses B Start at Day 5 menses C No additional contraceptive protection required D Require additional contraceptive protection and avoid sex for 7 days E Require additional contraceptive protection and avoid sex for 9 days F Discard the pills and restart back from day 1 menses G Discard the pills, induce bled with progesterone and take the pill at day 1 menses H Avoid sex for 7 days and condom I Continue pill and contraception for 7 days ( barrier method ) J Offer levonorgestrel 1.5mg stat
  • 29. 25. A 27-year-old healthy lady had first trimester abortion and she took the COCP at day 5 post partum abortion. 26. A 32-year-old healthy lady had recently second trimester miscarriage and she took COCP at day 20 post abortion. Her UPT test done negative. 27. A 30-year-old healthy lady had her menses on day 40 post partum. She took estradiol validate as her choice of contraception on the day itself. She presented to you for advice regarding contraception covers as she is now worried after an SI last night.